SlideShare una empresa de Scribd logo
1 de 31
CHAPTER 2

                                            FOURIER SERIES
PERIODIC FUNCTIONS
     A function f (x ) is said to have a period T if for all x, f ( x + T ) = f ( x) , where T is a

positive constant. The least value of T>0 is called the period of f (x) .


EXAMPLES
          We know that f (x ) = sin x = sin (x + 4 π ) = … Therefore the function has period 2 π , 4
π , 6 π , etc. However, 2 is the least value and therefore is the period of f(x).
         Similarly cos x is a periodic function with the period 2 π and tan x has period π .


DIRICHLET’S CONDITIONS
        A function f (x ) defined in c ≤ x ≤ c+2l can be expanded as an infinite trigonometric
                   a               nπx              nπx
series of the form o + ∑ a n cos        + ∑ bn sin      , provided
                    2               l                 l
        1. f (x) is single- valued and finite in (c , c+2l)
         2. f (x) is continuous or piecewise continuous with finite number of finite
             discontinuities in (c , c+2l).
         3. f (x) has no or finite number of maxima or minima in (c , c+2l).


EULER’S FORMULAS
         If a function f (x) defined in (c , c+2l) can be expanded as the infinite trigonometric

         ao     ∞
                           nπx ∞         nπx
series
         2
            +   ∑ an cos
                n =1        l
                              + ∑ bn sin
                                n =1      l
                                             then

                                c + 2l
                            1                         nπx
                       an =
                            l     ∫
                                  c
                                         f ( x) cos
                                                       l
                                                          dx, n ≥ 0

                                c + 2l
                            1                         nπx
                       bn =
                            l     ∫c
                                         f ( x) sin
                                                       l
                                                          dx, n ≥ 1


[ Formulas given above for a n and bn are called Euler’s formulas for Fourier coefficients]
DEFINITION OF FOURIER SERIES
                                                 ao     ∞
                                                                   nπx ∞         nπx
             The infinite trigonometric series
                                                 2
                                                    +   ∑ an cos
                                                        n =1        l
                                                                      + ∑ bn sin
                                                                        n =1      l
                                                                                     is called the

Fourier series of f (x) in the interval c ≤ x ≤ c+2l, provided the coefficients are given by the
Euler’s formulas.
EVEN FUNCTION
            If f (x) = φ (x) in (-l , l) such that φ (− x) = φ (x) , then f (x ) is said to be an even
function of x in (-l , l).
                        φ1 ( x ) in (−l ,0)
            If f ( x) = 
                        φ 2 ( x) in (0, l )
Such that φ1 (− x) = φ 2 ( x) or φ 2 ( − x ) = φ1 ( x ) , then f (x) is said to be an even function of x in
(-l , l).
EXAMPLE
    y = cos x , y = x 2 are even functions.

ODD FUNCTION
            If f (x) = φ (x) in (-l , l) such that φ (− x) = - φ (x) , then f (x) is said to be an odd
function of x in (-l , l).
                        φ1 ( x ) in (−l ,0)
            If f ( x) = 
                        φ 2 ( x) in (0, l )
Such that φ1 (− x) = - φ 2 ( x ) or φ 2 ( − x ) = - φ1 ( x ) , then f (x) is said to be an odd function of x in
(-l , l).
EXAMPLE
    y = sin x , y = x are odd functions.

FOURIER SERIES OF EVEN AND ODD FUNCTIONS
            1. The Fourier series of an even function f (x ) in (-l , l) contains only cosine terms




                                                                                                              2
(constant term included), i.e. the Fourier series of an even function f (x) in (-l , l) is
            given by
                                          ao                         nπx
                           f (x) =
                                          2
                                             +       ∑a    n   cos
                                                                      l
                                                                         ,

                                      nπx
                            l
                       2
            where a n = ∫ f ( x ) cos     dx.
                       l 0             l

       2. The Fourier series of an odd function f (x) in (-l , l) contains only sine terms, i.e.

            the Fourier series of an odd function f (x ) in (-l , l) is given by
                                                           nπx
                           f (x) =        ∑b     n   sin
                                                            l
                                                               ,

                                       nπx
                           l
                         2
            where bn =     ∫ f ( x) sin l dx.
                         l 0

PROBLEMS
1. Find the Fourier series of period 2l for the function f (x ) = x(2l – x) in (0 , 2l). Deduce

                         1   1   1
the sum of f (x) =       2
                           − 2 + 2 −
                        1   2   3
Solution:
                                   ao     ∞
                                                           nπx ∞         nπx
                 Let f (x ) =
                                   2
                                      +   ∑a
                                          n =1
                                                 n   cos
                                                            l
                                                              + ∑ bn sin
                                                                n =1      l
                                                                             in (0 , 2l)      …………(1)

                                                 nπx
                                  2l
                                1
                        an =
                                l0∫ x(2l − x) cos l dx
                                                                                                         2l
                                          nπx                       nπx               nπx 
                                      sin                    − cos             − sin      
                          1         2        l  − (2l − 2 x )         l  + (−2)         l  ,
                         = (2lx − x )
                          l           nπ                     n 2π 2            n 3π 3 
                                                                                          
                                          l                      l2                l3      0
                                                                                  using Bernoulli’s formula.
                                                                    2
                                   1
                          =            [ − 2l cos 2nπ − 2l ] = − 4l 2
                                n 2π 2                          n 2π
                                                                             2l
                                             1      x3 
                              2l
                            1                             4
                       a o = ∫ x(2l − x )dx = lx 2 −  = l 2 .
                            l0               l      3 0 3




                                                                                                               3
nπx
                                        2l
                                     1
                             bn =
                                     l0∫ x(2l − x) sin l dx
                                   =0
  Using these values in (1), we have

                                     2 2 4l 2                     ∞
                                                                         1          nπx
                         x (2l - x) = l − 2
                                     3   π
                                                                  ∑n
                                                                  n =1
                                                                         2
                                                                             cos
                                                                                     l
                                                                                        in (0, 2l)   ……………..(2)

                                              1   1   1
        The required series                   2
                                                − 2 + 2 − … ∞ can be obtained by putting x = l in the Fourier
                                             1   2   3
series in (2).
        x = l lies in (0 , 2l) and is a point of continuity of the function f (x) = x(2l – x).
                 ∴   [   Sum the Fourier series in (2)                        ] x =1   = f(l)
                                             ∞
                      2 2 4l 2                         1
                 i.e. l − 2
                      3   π
                                            ∑n
                                            n =1
                                                       2
                                                           cos nπ = l(2l - l)

                           4l 2    1     1   1         l
                                                           2
                 i.e.. -           − 2 + 2 − 2 + ...∞  =
                           π2      1    2   3          3

                 ∴
                          1   1   1        π2
                            − 2 + 2 − …∞ =
                         12 2    3         12


2. Find the Fourier series of period 2 π for the function f (x) = x cos x in 0 < x < 2 π .
Solution:
                                                           ∞                    ∞
                                             ao
                     Let f (x ) =
                                             2
                                                +          ∑ an cos nx + ∑ bn sin nx
                                                           n =1                n =1
                                                                                                     .……..…………(1)

                                            2π
                                        1
                             an =
                                        π    ∫ x cos x cos nxdx
                                             0

                                                 2π
                                         1
                                    =
                                        2π         ∫ x[ cos(n + 1) x + cos(n − 1) x]dx
                                                   0


                                       1          sin( n + 1) x cos(n + 1) x  2π  sin( n − 1) x cos(n − 1) x  2π 
                                    =             x.           +              +  x.            +              ,
                                      2π             n +1        (n + 1) 2  0        n −1        ( n − 1) 2  0 
                                                                                                                     
                                                                                                          if n ≠ 1
                                    =0,            if n ≠ 1
                                  ao = 0


                                                                                                                     4
2π                       2π
                             1                      1
                        an =         ∫ x cos xdx = 2π        ∫ x(1 + cos 2 x)dx
                                                  2

                             π       0                       0
                                                                           2π
                               1        x   sin 2 x cos 2 x 
                                             2
                            =            +x        +         = π.
                              2π        2      2      4 0
                                    2π
                                1
                        bn =
                                π   ∫ x cos x sin nxdx
                                    0



                                        2π
                               1
                            =
                              2π         ∫ x[ sin(n + 1) x + sin(n − 1) x]dx
                                         0


                               1         − cos(n + 1) x sin( n + 1) x  2π  − cos(n − 1) x sin( n − 1) x  2π 
                            =            x.            +               +  x.             +               ,
                              2π             n +1         (n + 1) 2  0         n −1         (n − 1) 2  0 
                                                                                                                
                           if n ≠ 1
                                      1    1      1       1     2n
                           =−           −     = −      +     =− 2   ,             if n ≠ 1
                                    n +1 n −1     n + 1 n − 1  n −1
                                    2π                            2π
                                1                             1
                         b1 =
                                π   ∫ x cos x sin xdx =
                                    0
                                                             2π   ∫ x sin 2 xdx
                                                                  0

                                                                      2π
                              1        − cos 2 x  sin 2 x    1
                           =          x          +         =−2
                             2π           2          4 0

Using these values in (1), we get
                                                     ∞
                                       1                          n
                       f(x) = π cos x − sin x − 2 ∑                  sin nx
                                                 n = 2 , 3,... n − 1
                                                                2
                                       2


3. Find the Fourier series expansion of f (x) = sin ax in (-l , l).
Solution:
             Since f (x) is defined in a range of length 2l, we can expand f (x ) in Fourier series of
period 2l.
             Also f ( − x) = sin[a(-x)] = -sin ax = - f (x)
         ∴ f (x) is an odd function of x in (-l , l).

 Hence Fourier series of f (x ) will not contain cosine terms.
                                     ∞
                                                       nπx
                  Let f (x ) =      ∑b
                                    n =1
                                             n   sin
                                                        l
                                                                                    ………………….(1)




                                                                                                            5
1   nπ                nπ
                                  l
                                                                
                             =     ∫ cos l − a  − cos l + a  xdx
                                 l 0                         
                                                                         l
                                  nπ                nπ     
                              1  sin  l − a  x sin  l + a  x 
                             =               −             
                              l       nπ              nπ
                                          −a               +a 
                                       l               l         
                                                                 0
                                 1            nπ            1         nπ      
                            =            sin      − a l −        sin      + a l
                              nπ − la  l               nπ + la  l             
                            =
                                 1
                              nπ − al
                                         {− (−1) n sin al} − nπ 1 al {(−1) n sin al}
                                                                +
                                                1             1 
                            = (−1) n +1 sin al           +        
                                                nπ − al nπ + al 
                              (−1) n +1 2nπ sin al
                            =
                                n 2π 2 − a 2 l 2
Using these values in (1), we get
                                   ∞
                                           (−1) n +1 n       nπx
               sin ax = 2π sin al ∑                      sin
                                  n =1   n π −a l
                                          2 2        2 2
                                                              l

4. Find the Fourier series expansion of f (x) = e − x in (−π , π ) . Hence obtain a series for
cosec π
Solution:
          Though the range (−π , π ) is symmetric about the origin, e − x is neither an even function
nor an odd function.
                                             ∞               ∞
                                      ao
∴                   Let f (x) =
                                      2
                                         +   ∑ an cos nx + ∑ bn sin nx
                                             n =1            n =1
                                                                                       ..…..…………(1)

in (−π , π ) [ the length of the range is 2π ]




                                                                                                      6
π
                                    1
                                      ∫π e cos nxdx
                                          −x
                            an =
                                    π−
                                                                                 π
                                 1  e −x                         
                                =  2     ( − cos nx + n sin nx ) 
                                 π n +1                           −π

                                =−
                                          1
                                                    {e   −π
                                                              (−1) n − e π (−1) n }
                                     π ( n + 1)
                                          2


                                     2( −1) n
                                =               sinh π
                                    π (n 2 + 1)


                                    2 sinh π
                            ao =
                                        π

                                      π
                                    1
                                      ∫π e sin nxdx
                                          −x
                            bn =
                                    π−
                                                                                π
                                 1  e −x                         
                                =  2     ( − sin nx − n cos nx ) 
                                 π n +1                           −π

                                =−
                                          n
                                                    {e   −π
                                                              (−1) n − e π (−1) n }
                                     π ( n 2 + 1)
                                    2n(−1) n
                                =               sinh π
                                    π (n 2 + 1)
Using these values in (1), we get
        sinh π 2 sinh π     ∞
                                (−1) n       2 sinh π                ∞
                                                                         (−1) n n
e−x =
          π
              +
                   π
                           ∑ n 2 + 1 cos nx + π
                           n =1
                                                                    ∑ n 2 + 1 sin nx
                                                                    n =1
                                                                                       in (−π , π )

[ Sum of   the Fourier series of f ( x )] x =0 = f (0),

                                              [Since x=0 is a point of continuity of f(x)]
              sinh π         ∞
                                  (−1) n 
i.e.,                  1 + 2∑ 2              −0
                                          = e =1
                π           n =1 n + 1

                                 −1   ∞
                                           (−1) n
i.e.,        π cos ech π = 1 + 2  + 2∑ 2
                                 2   n=2 n + 1


                            2 ∞ (−1) n
i.e.,         cos ech π =     ∑
                            π n=2 n 2 + 1




                                                                                                      7
HALF-RANGE FOURIER SERIES AND PARSEVAL’S THEOREM
    (i) The half range cosine series in (0 , l) is
                                                         ao     ∞
                                                                                 nπx
                                      f (x) =
                                                         2
                                                            +   ∑a
                                                                n =1
                                                                       n   cos
                                                                                  l
                                                     l
                                                   2
                                                   l∫
                           where a o =                 f ( x )dx.
                                                     0


                                                                 nπx
                                                     l
                                                   2
                                         an =        ∫ f ( x) cos l dx.
                                                   l 0

    (ii) The half range sine series in (0 , l) is
                                  ∞
                                                   nπx
                       f (x) =    ∑b
                                  n =1
                                         n   sin
                                                    l
                                                       ,

                                          nπx
                              l
                            2
              where bn =      ∫ f ( x) sin l dx.
                            l 0

    (iii) The half range cosine series in (0 , π ) is given by


                                                                ∞
                                                         ao
                                      f (x) =
                                                         2
                                                            +   ∑a
                                                                n =1
                                                                       n   cos nx

                                                     π
                                      2
                           where a o = ∫ f ( x )dx.
                                      π 0
                                                     π
                                                   2
                                                   π∫
                                         an =          f ( x ) cos nxdx.
                                                     0


    (iv) The half range sine series in (0 , π ) is given by


                                  ∞
                       f (x) =    ∑b
                                  n =1
                                         n   sin nx ,

                              π
                        2
              where bn = ∫ f ( x) sin nxdx.
                        π 0




                                                                                       8
ROOT-MEAN SQUARE VALUE OF A FUNCTION
Definition
                                                                                                                   c +2 l
                                                                                                              1
                                                                                                                     ∫y
                                                                                                                            2
           If a function y = f (x ) is defined in (c , c+2l), then                                                              dx is called the root mean-
                                                                                                              2l     c


square(R.M.S.) value of y in (c , c+2l) and is denoted by y.
                                                      c + 2l
                                  1 2
                         Thus y =                       ∫y
                                                               2
                                                                   dx.
                                  2l                    c


PARSEVAL’S THEOREM
           If y = f (x ) can be expanded as a Fourier series of the form
ao        ∞
                              nπx ∞         nπx
2
   +     ∑ an cos
         n =1                  l
                                 + ∑ bn sin
                                   n =1      l
                                                in (c , c+2l), then the root-mean square value y of y = f (x)

in       (c , c+2l) is given by
                               1 2 1 ∞        1 ∞
                            y = a o + ∑ a n + ∑ bn
                             2              2        2

                               4     2 n =1   2 n =1
PROOF
                                 ao      ∞
                                                               nπx ∞         nπx
                f (x) =
                                 2
                                    +   ∑ an cos
                                         n =1                   l
                                                                  + ∑ bn sin
                                                                    n =1      l
                                                                                 in (c , c+2l)                                       ....……………….(1)

∴ By Euler’s formulas for the Fourier coefficients,
                                                                        c + 2l
                                                             1                                nπx
                                                        an =
                                                             l            ∫ c
                                                                                 f ( x) cos
                                                                                               l
                                                                                                  dx, n ≥ 0                         ..…………………(2)

                                                                        c + 2l
                                                                    1                         nπx
                                                        bn =
                                                                    l     ∫
                                                                          c
                                                                                 f ( x) sin
                                                                                               l
                                                                                                  dx, n ≥ 1                        …....……………..(3)

Now, by definition,
                   c + 2l                    c + 2l
              1                         1
                                                ∫ [ f ( x)]
     2
                     ∫ y dx =
                                                                   2
     y =                2
                                                                       dx
              2l     c
                                        2l      c

                   c + 2l
           1                       a    ∞
                                                   nπx ∞         nπx 
         =           ∫      f ( x)  o + ∑ a n cos    + ∑ bn sin       dx,                                               using (1)
           2l        c              2 n =1         l   n =1      l 
           ao1 c + 2l          ∞ a 1 c + 2l        nπx  ∞ bn 1
                                                                          c + 2l
                                                                                 nπx 
         =    ∫      f ( x)dx  + ∑ n  ∫ f ( x) cos    dx  + ∑  ∫ f ( x) sin    dx 
           4  l c                n =1 2  l c        l       n =1 2  l c      l    
                        ∞             ∞
          ao                an            bn
         = .a o + ∑ .a n + ∑ .bn ,                       by using (2) and (3)
          4            n =1 2        n =1 2




                                                                                                                                                          9
∞2      ∞     2            2
        ao      an      b
      =    +∑      +∑ n .
         4  n =1 2  n =1 2

EXAMPLES
1. Find the half-range (i) cosine series and (ii) sine series for f (x ) = x 2 in (0 , π )
Solution:
           (i) To get the half-range cosine series for f (x ) in (0 , π ), we should give an even

extension for f (x) in ( − π , 0).

      i.e. put f (x) = ( − x ) 2 = x 2 in ( − π , 0)

Now f (x) is even in ( − π , π ).
                                                       ∞
                                                ao
                  ∴            f (x) =
                                                2
                                                   +   ∑a
                                                       n =1
                                                              n   cos nx          ………………….(1)

                                            π
                                       2
                               an =
                                       π    ∫ f ( x) cos nxdx.
                                            0


                                            π
                                       2 2
                                       π∫
                                  =       x cos nxdx
                                        0

                                                                                  π
                                   2   sin nx        − cos nx   − sin nx 
                                  = x2         − 2 x           + 2       
                                   π  n              n
                                                             2
                                                                    n
                                                                          3
                                                                                0
                                        4               4(−1) n
                                  =         .π (−1) n =         ,n ≠ 0
                                       πn 2               n2
                                            π                      π
                                       2              2 2        2 2
                               ao =      ∫ f ( x)dx = π ∫ x dx = 3 π
                                       π 0              0

∴ The Fourier half-range cosine series of x 2 is given by

                                       π2     ∞
                                                 (−1) n
                                x2 =      + 4∑ 2 cos nx in (0 , π ).
                                       3     n =1 n


      (ii) To get the half-range sine series of f (x ) in (0 , π ), we should give an odd extension

for f (x) in (- π , 0).

    i.e.              Put f (x ) = - ( − x ) 2 in (- π , 0)

                                = - x 2 in (- π , 0)
Now f (x) is odd in (- π , π ).




                                                                                                      10
∞
     ∴                  f (x) =   ∑b
                                  n =1
                                          n   sin nx                                          ……………….(2)

                                  π                                      π
                             2                   2
                         bn = ∫ f ( x) sin nxdx = ∫ x 2 sin nxdx
                             π 0                 π 0
                                                                                         π
                              2   cos nx        sin nx   cos nx 
                             = x2  −      − 2 x −       + 2     
                              π      n            n 2   n 3  0  
                               2 π 2                      
                                   (−1) + 3 {(−1) − 1} 
                                        n +1 2
                             =                        n

                               π n          n             
                                2 π 2 4 
                                     − , if n is odd
                             = π  n n 3 
                               − 2π ,         if n is even
                                n
Using this value in(2), we get the half-range sine series of x 2 in (0 , π ).


2.     Find the half-range sine series of f (x) = sin ax in (0 , l).
Solution:
       We give an odd extension for f (x) in (-l , 0).
     i.e. we put f (x) = -sin[a(-x)] = sin ax in (-l , 0)
     ∴     f (x) is odd in (-l , l)
                                                  ∞
                                                                   nπx
     Let                       f (x ) =           ∑b
                                                  n =1
                                                         n   sin
                                                                    l

                                                         nπx
                                              l
                                          2
                                 bn =       ∫ sin ax. sin l dx
                                          l 0
                                          1   nπ                  nπ
                                              l
                                                                           
                                      =     ∫ cos l − a  x − cos l + a  x dx
                                          l 0                           
                                                                                  l
                                            nπ                     nπ       
                                        1  sin  l − a  x sin  l + a  x 
                                      =                  −                  
                                        l   nπ                    nπ       
                                            l − a                     + a 
                                                                  l        0
                                             1                                   1
                                      =           (−1) n +1 sin ( nπ − al ) −         sin ( nπ + al )
                                        nπ − al                               nπ + al




                                                                                                           11
1                             1
                                  =            (−1) n +1 sin al +         ( −1) n +1 sin al
                                    nπ − al                       nπ + al
                                                           2nπ
                                  = (−1) n +1 sin al. 2 2
                                                      n π − a 2l 2
Using this values in (1), we get the half-range sine series as
                                              ∞
                                                   (−1) n +1 .n       nπx
                          sin ax = 2π sin al ∑ 2 2                sin
                                             n =1 n π − a l
                                                              2 2
                                                                       l
3.    Find the half-range cosine series of f (x ) = a in (0 , l). Deduce the sum of

      1   1   1
      2
        + 2 + 2 + ∞ .
     1   3   5
 Solution:
     Giving an odd extension for f (x) in (-l , 0), f (x ) is made an odd function in (-l , l).
                                                          nπx
        ∴ Let                  f(x) =    ∑b     n   sin
                                                           l
                                                                                              ..……………(1)

                                               nπx
                                           l
                                      2
                                  bn = ∫ a sin     dx
                                      l 0       l
                                                                 l
                                                   nπx 
                                             − cos l 
                                       =
                                         2a 
                                             nπ
                                          l 
                                                        
                                                         =
                                                            2a
                                                            nπ
                                                               1 − ( − 1)
                                                                          n
                                                                           {      }
                                                        
                                            
                                                 l     0
                                                        

                                           4a
                                           ,               if n is odd
                                        =  nπ
                                          0,
                                                           if n is even
     Using this value in (1), we get
                                               4a ∞      1   nπx
                                      a=            ∑5 n sin l in (0 , l )
                                               π n =1,3,

     Since the series whose sum is required contains constant multiples of squares of bn , we apply
Parseval’s theorem.
                              l
                1        1
                  ∑ bn = l ∫ [ f ( x)] dx
                      2               2

                2          0




                                                                                                           12
∞
                     1 16a 2                     1
             i.e.     .
                     2 π2
                                  ∑ ( 2n − 1)
                                n =1, 3, 5
                                                         2
                                                             = a2

                                          ∞
                               8a 2              1
             i.e.
                               π2
                                          ∑ ( 2n − 1)
                                          n =1
                                                         2
                                                             = a2

                                          ∞
                                             1       π2
             ∴                        ∑ ( 2n − 1) 2 8 .
                                      n =1
                                                   =

4. Expand f (x) = x - x 2 as a Fourier series in -1 < x < 1 and using this series find the

   r.m.s. value of f (x ) in the interval.
Solution:
     The Fourier series of f (x ) in (-1 , -1) is given by
                                           ∞                    ∞
                              ao
                    f (x) =
                              2
                                 +        ∑ an cos nπx + ∑ bn sin nπx
                                          n =1                  n =1
                                                                                         .………………(1)

                                  1                  1
                         a o = ∫ f ( x)dx = ∫ ( x − x 2 ) dx
                              1
                              1 −1          −1
                                                 1
                               x2 x3    1 1  1 1
                            =  −  =  − − + 
                               2     
                                  3  −1  2 3   2 3 
                              −2
                         ao =                                                              ..........................(2)
                                3
                         1                               1

                        ∫1 f ( x) cos nπx dx = −∫1( x − x ) cos nπx dx
                      1
               an =                                      2

                      1−
                                                                                               1
                                   sin nπx                − cos nπx         − sin nπx 
                    = ( x − x 2 )           − (1 − 2 x )       2     + (−2)      3    
                                   n                         n              n          −1
                         − cos nπ 3 cos nπ
                     =           −
                            n2       n2
                              4 cos nπ
                    an = −                                                              ……………….(3)
                                 n2




                                                                                                                     13
1                       1

              ∫1 f ( x) sin nπx dx = −∫1( x − x ) sin nπx dx
            1
     bn =                                      2

            1−
                                                                                        1
                        − cos nπx                − sin nπx         cos nπx 
         = ( x − x 2 )             − (1 − 2 x )             + (−2) 3 3 
                         nπ                        nπ                 n π  −1
                                                         2 2
                                                             
           − 2 cos nπ 2 cos nπ 2 cos nπ
         =                 −           + 3 3
               n 3π 3           nπ          nπ
                   n +1
            2(−1)
      bn =                                                                          ..........................( 4)
               nπ
Substituting (2), (3), (4) in (1) we get
                            1 ∞ 4(−1) n +1                 ∞
                                                               2(−1) n +1
                 f (x) = − + ∑                  cos nπx + ∑               sin nπx
                            3 n =1 n 2                    n =1   nπ
We know that r.m.s. value of f(x) in (-l , l) is
                             1 2 1 ∞         1 ∞
                               a o + ∑ a n + ∑ bn
                        2                  2        2
                       y =                                                                   ……………….(5)
                             4      2 n =1   2 n =1
From (2) we get
                             −2     2  4
                      ao =      ⇒ ao =                                                      .………………..(6)
                              3        9
From (3) we get
                             4( −1) n +1     2 16
                      an =        2
                                         ⇒ an = 4                                           ………………..(7)
                                n              n
From (4) we get
                             2(−1) n +1     2   4
                      bn =              ⇒ bn = 2 2                                           ..………………(8)
                               nπ             nπ
Substituting (6), (7) and (8) in (5) we get
                             1 1 ∞  16    4 
                              + ∑ 4 + 2 2 
                        2
                       y =
                             9 2 n =1  n nπ 
5. Find the Fourier series for f (x) = x 2 in − π < x < π . Hence show that

 1   1   1      π4
   + 4 + 4 + =
14 2    3       90
Solution:
      The Fourier series of f (x ) in (-1 , 1) is given by

                                                      π2         ∞
                                                                     4(−1) n
                                              f (x) =
                                                       3
                                                         +      ∑ n 2 cos nx
                                                                n =1




                                                                                                                     14
The co-efficients a o , a n , bn are

                                                            2π 2        4(−1) n
                                                ao =             , an =         , bn = 0
                                                             3            n2
Parseval’s theorem is

                                                                               (a                      )
                               π                                        ∞
                        1                      1 2 1
                               ∫ [ f ( x)] dx = ao + ∑
                                          2                                             2          2
                                                                                    n       + bn
                       2π      −π
                                                              4     2   n =1

                                                             ao 2 1 ∞        2 
                                                                                (                          )
                                    π

                                    ∫ [x ]                        + ∑ a n + bn 
                                          2 2
                                                    dx = 2π 
                                                                          2
                      ∴
                                    −π                       4
                                                                  2 n =1       
                                                                                
                                                        π
                                          x5              π 4 1 ∞ 16 
                       i.e.,             
                                          5         = 2π 
                                                               + ∑ 4
                                                    −π    9 2 n =1 n 
                                   2π 5 2π 5    ∞
                                                    16
                       i.e.,           −     =π∑ 4
                                    5    9     n =1 n


                                                8π 4   ∞
                                                           16
                                                     =∑ 4
                                                 45   n =1 n
                                                ∞
                                                  1 π4
                       i.e.,                 ∑ n 4 = 90
                                             n =1


                     1   1   1        π4
          i.e.,        + 2 + 2 + ∞ =
                    12 3    5         90

HARMONIC ANALYSIS
        The process of finding the Fourier series for a function given by numerical value is
known as harmonic analysis. In harmonic analysis the Fourier coefficients ao , a n , and bn of the
function y = f (x) in (0 , 2 π ) are given by
             a o = 2[mean value of y in (0 , 2 π )]

                  a n = 2[mean value of y cos nx in (0 , 2 π )]

                  bn = 2[mean value of y sin nx in (0 , 2 π )]

(i) Suppose the function f (x) is defined in the interval (0 , 2l), then its Fourier series is,
                               ao        ∞
                                                              nπx ∞         nπx
                  f (x) =
                               2
                                  +      ∑a
                                         n =1
                                                    n   cos
                                                               l
                                                                 + ∑ bn sin
                                                                   n =1      l

   and now, a o = 2[mean value of y in (0 , 2l)]

                                              nπx             
                  a n = 2 mean value of y cos     in (0 , 2l )
                                               l              




                                                                                                               15
                    nπx             
               bn = 2 mean value of y sin     in (0 , 2l )
                                           l              
(ii) If the half range Fourier sine series of f (x) in (0 , l) is,
                          ∞
                                            nπx
               f (x) =   ∑b
                         n =1
                                 n   sin
                                             l
                                                , then

                                          nπx            
               bn = 2 mean value of y sin     in (0 , l )
                                           l             
(iii) If the half range Fourier sine series of f (x) in (0 , π ) is,
                           ∞
                                            nπx
               f (x) =    ∑b
                          n =1
                                 n    sin
                                             l
                                                , then


                bn = 2[ mean value of y sin nx in (0 , π )]

(iv) If the half range Fourier cosine series of f (x) in (0 , l) is,
                        ao    ∞
                                       nπx
                f (x) =    + ∑ a n cos       , then
                         2 n =1         l
                a o = 2[mean value of y in (0 , l)]

                                            nπx            
                a n = 2 mean value of y cos     in (0 , l )
                                             l             
(v) If the half range Fourier cosine series of f (x) in (0 , π ) is,
                         ao          ∞
                                                      nπx
               f (x) =
                         2
                            +        ∑a
                                     n =1
                                            n   cos
                                                       l
                                                          , then

                a o = 2[mean value of y in (0 , π )]

                a n = 2[ mean value of y cos nx in (0 , π )] .

EXAMPLES
1. The following table gives the variations of a periodic function over a period T.
                                      0               T             T      T           2T      5T      T
                   x                                  6              3     2            3       6
                 f (x)           1.98                 1.3          1.05   1.3         -0.88   -0.25   1.98
                                                                                2πx
Show that f (x ) = 0.75 + 0.37 cos θ +1.004 sin θ , where θ =
                                                                                 T
Solution:
       Here the last value is a mere repetition of the first therefore we omit that value and
consider the remaining 6 values. ∴ n = 6.


                                                                                                             16
2πx
               Given          θ=                                                 ..………………..(1)
                                    T
                                               T T T 2T 5T                                π 2π
     ∴ when x takes the values of 0,            , , ,  ,          θ takes the values 0,    ,   ,
                                               6 3 2 3   6                                3 3

     4π       5π
π,        ,      . (By using (1))
      3        3
Let the Fourier series be of the form
                              ao
                   f ( x) =      + a1 cos θ + b1 sin θ ,                            ………………(2)
                              2
                               ∑y
          where         a o = 2   ,
                                n 
                                  
                                  ∑ y cos θ 
                           a1 = 2           ,
                                     n      
                                            
                                  ∑ y sin θ 
                           b1 = 2           ,       n=6
                                     n      
                                            
              θ                   y           cos θ        sin θ          y cos θ          y sin θ
              0°                1.98           1.0           0             1.98               0
              π
                                1.30          0.500        0.866           0.65           1.1258
               3




          2π 3                  1.05         -0,500        0.866          -0.525          0.9093
           π                    1.30            -1           0             -1.3               0
          4π 3                -0.88          -0.500       -0.866           0.44            0.762
          5π 3                -0.25           0.500       -0.866          -0.125           0.2165
                                 4.6                                       1.12            3.013
                                     ∑y
                            a o = 2      = 1.5, a1 = 2 ∑ y cos θ = 0.37
                                      6              6
                                        
                                   2
                            b1 = ∑ y sin θ = 1.00456
                                   6
Substituting these values of a o , a1 , and b1 in (2), we get
                           ∴ f (x) = 0.75 + 0.37 cos θ + 1.004 sin θ

2. Find the Fourier series upto the third harmonic for the function y = f (x) defined in
     (0 , π ) from the table
                       x             0            π         2π   3π         4π        5π             π
                                                  6          6    6          6         6



                                                                                                         17
f (x)       2.34         2.2           1.6          0.83      0.51        0.88         1.19
   Solution:
       We can express the given data in a half range Fourier sine series.
                 f ( x) = b1 sin x + b2 sin 2 x + b3 sin 3 x           ..………………...(1)
               x      y = f(0)      sin x     sin 2x       sin 3x    y sin x    y sin 2x      y sin 3x
             0          2.34          0          0           0          0             0           0
             30          2.2         0.5       0.87          1         1.1          1.91         2.2
             60          1.6        0.87       0.87          0       1.392         1.392          0
             90         0.83          1          0          -1        0.83            0        -0.83
            120         0.51        0.87      -0.87          0        0.44         -0.44          0
            150         0.88         0.5      -0.87          1        0.44          0.76        0.88
            180         1.19          0          0           0          0             0           0
                                                                     4.202         3.622        2.25
                               ∑ y sin x  1
       Now             b1 = 2             = [ 4.202] = 1.40
                              
                                  6       3
                                          
                               ∑ y sin 2 x  1
                       b2 = 2               = [ 3.622] = 1.207
                              
                                   6        3
                                            
                                ∑ y sin 3 x  1
                        b3 = 2               = [ 2.25] = 0.75
                               
                                    6        3
                                             
       Substituting these values in (1), we get


                        f (x) = 1.4 sin x + 1.21 sin 2x + 0.75 sin 3x
3. Compute the first two harmonics of the Fourier series for f(x) from the following data

              x              0         30            60            90        120        150           180
            f (x )           0        5224          8097          7850      5499       2626            0




Solution:
      Here the length of the interval is π . ∴ we can express the given data in a half range
Fourier sine series
       i.e.,           f ( x) = b1 sin x + b2 sin 2 x                               ………………………(1)



                                                                                                                   18
x                 y            sin x            sin 2x
                   0                 0                0               0
                  30              5224               .5             0.87
                  60              8097             0.87             0.87
                  90              7850                1               0
                 120              5499             0.87            -0.87
                 150              2626              0.5            -0.87
                               ∑ y sin x 
       Now              b1 = 2            = 7867.84
                              
                                  6      
                                          
                                  ∑ y sin 2 x 
                          b2 = 2               = 1506.84
                                 
                                      6       
                                               
                 ∴ f (x) = 7867.84 sin x + 1506.84 sin 2x
4. Find the Fourier series as far as the second harmonic to represent the function given in
   the following data.
                   x             0           1          2          3            4          5
                 f (x )          9          18         24         28           26         20
Solution:
    Here the length of the interval is 6 (not 2 π )
       i.e., 2l = 6 or l = 3
 ∴ The Fourier series is

                       ao         πx         2πx         πx        2πx
            f ( x) =      + a1 cos + a 2 cos     + b1 sin + b2 sin                  …………………..(1)
                       2           3          3           3         3


                  πx             2πx                         πx           πx            2πx            2πx
                                                     y cos        y sin         y cos          y sin
      x            3              3          y                3            3             3              3
     0           0               0           9           9          0                 9           0
     1           π 3            2π 3       18           9          15.7             -9           15.6
     2          2π 3            4π 3        24         -12         20.9             -24           0
     3            π              2π         28         -28          0                28           0
     4          4π 3            8π 3        26         -13        -22.6             -13          22.6
     5          5π 3           10 π 3       20          10        -17.4             -10         -17.4
                                           125         -25         -3.4             -19          20.8




                                                                                                             19
 ∑ y  2(125)
                               Now a o = 2     =        = 41.66,
                                           6        6
                                               
                                         2         πx
                                   a1 = ∑ y cos = −8.33
                                         6          3
                                         2        πx
                                   b1 = ∑ y sin       = −1.13
                                         6         3
                                                                  2        2πx
                                                  a2 =
                                                                  6
                                                                    ∑ y cos 3 = −6.33
                                                              2        2πx
                                                 b2 =
                                                              6
                                                                ∑ y sin 3 = 6.9
Substituting these values of a o , a1 , b1 , a 2 and b2 in (1), we get
                        41.66                                     πx                     2πx                πx               2πx
             f ( x) =             − 8.33 cos                           − 6.33 cos              − 1.13 sin        + 6.9 sin
                           2                                      3                        3                3                3
COMPLEX FORM OF FOURIER SERIES
                                                                             ∞
        The equation of the form                           f ( x) =         ∑c e
                                                                         n = −∞
                                                                                  n
                                                                                      inπx l




is called the complex form or exponential form of the Fourier series of f (x) in (c , c+2l). The

coefficient c n is given by
                                  c + 2l
                          1
                                     ∫ f ( x )e
                                                            −inπx l
                     cn =                                              dx
                          2l         c


When l = π , the complex form of Fourier series of f (x) in (c , c+2 π ) takes the form
                                 ∞
                   f ( x) =    ∑c e
                               n = −∞
                                         n
                                                 inx
                                                       ,          where

                                        c + 2π
                              1
                                          ∫ f ( x )e
                                                              −inx
                        cn =                                           dx.
                             2π           c


PROBLEMS
1. Find the complex form of the Fourier series of f (x) = e x in (0 , 2).
Solution:
   Since 2l = 2 or l = 1, the complex form of the Fourier series is
                                         ∞
                         f ( x) =       ∑c e
                                     n = −∞
                                                  n
                                                           inπx




                                                                                                                                   20
2
                             1
                        c n = ∫ f ( x)e −inπx dx
                             20
                                  2
                            1
                           = ∫ e x e −inπx dx
                            20
                                                       2
                            1  e ( 1−inπ ) x 
                           =                 
                            2  1 − inπ  0

                           =
                                     1
                                2(1 − inπ )
                                            {e 2(1−inπ ) − 1}

                            =
                                  (1 + inπ )               {e ( cos 2nπ − i sin 2nπ ) − 1}
                                                              2

                                2(1 + n π     2    2
                                                       )
                            =
                                (e    2
                                       − 1)(1 + inπ )
                                      2(1 + n 2π 2 )
Using this value in (1), we get
                      e 2 − 1  ∞ (1 + inπ ) inπx
                      2  ∑ (1 + n 2π 2 ) e
                ex =          
                               n =−∞
2. Find the complex form of the Fourier series of f (x) = sin x in (0 , π ).
Solution:
     Here 2l = π or l = π 2 .
      ∴ The complex form of Fourier series is
                                                    ∞
                                  f ( x) =        ∑c e
                                                  n = −∞
                                                             n
                                                                  i 2 nx
                                                                                                      …………………..(1)

                                                        π
                                               1
                                          c n = ∫ sin xe −i 2 nx dx
                                               π 0
                                                                                                 π
                                                1           e −i 2 nx                           
                                              =                         { − i 2n sin x − cos x} 
                                                π          1 − 4n
                                                                       2
                                                                                                 0

                                              =
                                                        1
                                                  π ( 4n − 1)
                                                         2
                                                                           [
                                                              − e i 2 nx − 1 = −   ]   2
                                                                                 π ( 4n 2 − 1)
Using this value in (1), we get
                                          2 ∞     1
                        sin x = −            ∑ 4n 2 − 1 .e i 2nx
                                          π n =−∞
                                                                                in (0 , π )

3.   Find the complex form of the Fourier series of f (x) = e − ax in (-l , l).
Solution:


                                                                                                                     21
Let the complex form of the Fourier series be
                                            ∞
                         f ( x) =           ∑c e
                                        n = −∞
                                                     n
                                                         inπx l



                                                 l
                                   1
                              c n = ∫ f ( x)e −inπx l dx
                                   2l −l

                                                 l
                                     1
                                    = ∫ e − ax e −inπx l dx
                                     2l −l
                                             l
                                  1
                                 = ∫ e −( al +inπ ) x / l dx
                                  2l −l
                                                                           l
                                  1  e −( al +inπ ) x l                  
                                 =                                       
                                  2l  − ( al + inπ )                   l  −l

                                    =
                                             1
                                      2( al + inπ )
                                                                  [
                                                     e −( al +inπ ) − e ( al +inπ )    ]
                                    =
                                      2
                                             1
                                        ( al + inπ )
                                                                  [
                                                     e al (−1) n − e − al (−1) n           ]
                                                     [ e   ± inπ
                                                                      = cos nπ ± i sin nπ = (−1) n   ]
                                      sinh al (−1) n
                                    =
                                        al + inπ
                                      sinh al.( al − inπ ) (−1) n
                                    =
                                           a 2 l 2 + n 2π 2
Using this value in (1), we have
                                                              ∞
                                                                  (−1) n ( al − inπ ) inπx l
                              e − ax = sinh al               ∑ 22 2 2 e
                                                            n = −∞ a l + n π
                                                                                                     in (-l , l)

4. Find the complex form of the Fourier series of f (x) = cos ax in (- π , π ), where a is
   neither zero nor an integer.
Solution:
   Here 2l = 2 π or l = π .
   ∴ The complex form of Fourier series is
                                ∞
                   f ( x) =    ∑c e
                              n = −∞
                                        n
                                            inx
                                                                                                         ………………….(1)




                                                                                                                       22
π
                                   1
                                       ∫π cos ax.e
                                                     − inx
                          cn =                               dx
                                  2π   −
                                                                                 π
                            1  e −inx                               
                         =      2        { − in cos ax + a sin ax} 
                           2π  a − n   2
                                                                      −π
                         =
                                 1
                           2π ( a − n 2 )
                                 2
                                                  [
                                           e −inπ ( − in cos aπ + a sin aπ ) − e inπ ( − in cos aπ − a sin aπ )   ]
                                 1
                         =                (−1) n 2a sin aπ
                           2π ( a − n )
                                 2    2


Using this value in (1), we get

                                    a sin aπ     ∞
                                                          (−1) n inx
                         cos ax =
                                        π
                                               ∑
                                               n = −∞    a2 − n2
                                                                 e         in (- π , π ).



                                                                  UNIT 2
                                                             PART – A
    1. Determine the value of a n in the Fourier series expansion of f ( x) = x 3 in − π < x < π .

     Ans: f ( x) = x 3 is an odd function.
     ∴ an = 0

    2. Find the root mean square value of f ( x) = x 2 in the interval (0 , π ) .
    Ans:
         RMS Vale of f ( x ) = x 2 in (0 , π ) is
                     π               π                             π
                                              1  x5              
                     ∫ [x ]
           2 1           2 2       1
         y =                   dx = ∫ x 4 dx =                   
             π       0
                                   π 0        π 5
                                                
                                                                  
                                                                  0
                   1 π 5  π 4
               =           =
                   π5  5
                      

3. Find the coefficient b5 of cos 5 x in the Fourier cosine series of the function f ( x ) = sin 5 x in

the interval (0 , 2π )
Ans: Here f ( x) = sin 5 x
Fourier cosine series is




                                                                                                                      23
∞
                            ao
                  f (x) =
                            2
                               +   ∑a
                                    n =1
                                           n   cos nx , where




          π                                π
        2                      2
an =      ∫ f ( x) cos nx dx = π ∫ sin 5 x cos nx dx
        π 0                      0
              π
       2
    =
      2π      ∫ [ sin(5 + n) x + sin(5 − n) x] dx
              0
                                                    π
      − 1  cos(5 + n) x cos(5 − n) x 
    =                   +               =0
      π  5+n
                           5 − n 0  
               cos x, if 0 < x < π
4. If f ( x) =                      and f ( x) = f ( x + 2π ) for all x, find the sum of the Fourier
               50,    if π < x ≤ 2π

series of f (x ) at x = π .
Ans:          Here π is a point of discontinuity.
∴ The sum of the Fourier series is equal to the average of right hand and left hand limit of the
given function at x = π .


                                f (π − 0) + f (π + 0)
i.e.,              f (π ) =
                                          2
                                cos π + 50 49
                            =             =
                                     2      2
5. Find bn in the expansion of x 2 as a Fourier series in (−π , π ) .

Ans:                   bn = 0

Since f ( x) = x 2 is an even function in (−π , π ) .

6. If f (x) is an odd function defined in (-l , l) what are the values of a 0

Ans:                   a0 = 0

a n = 0 since f (x) is an odd function.

7. Find the Fourier constants bn for x sin x in (−π , π ) .

Ans:              bn = 0

Since f ( x) = x sin x is an even function in (−π , π ) .



                                                                                                   24
8. State Parseval’s identity for the half-range cosine expansion of f (x ) in (0 , 1).
Ans:
                 1                          2
                                            ∞
                                      a0
              2 ∫ [ f ( x)] dx =         + ∑ an
                            2                   2

                 0
                                       2   n =1


where
                                        1
                                a 0 = 2 ∫ f ( x) dx
                                        0
                                        1
                                a n = 2 ∫ f ( x) cos nx dx
                                        0


9. Find the constant term in the Fourier series expansion of f ( x ) = x in (−π , π ) .
Ans:
           a 0 = 0 since f (x ) is an odd function in (−π , π ) .

10. State Dirichlet’s conditions for Fourier series.
Ans:
(i)    f (x) is defined and single valued except possibly at a finite number of points in (−π , π ) .

(ii) f (x) is periodic with period 2 π .
(iii) f (x) and f ′(x) are piecewise continuous in (−π , π ) .
      Then the Fourier series of f (x ) converges to
           (a) f (x) if x is a point of continuity
                     f ( x + 0) + f ( x − 0)
           (b)                               if x is a point of discontinuity.
                                2
11. What you mean by Harmonic Analysis?
Ans:
         The process of finding the Fourier series for a function given by numerical value is

known as harmonic analysis. In harmonic analysis the Fourier coefficients ao , a n , and bn of the

function y = f (x) in (0 , 2 π ) are given by


                 a o = 2[mean value of y in (0 , 2 π )]

                 a n = 2[mean value of y cos nx in (0 , 2 π )]

                 bn = 2[mean value of y sin nx in (0 , 2 π )]



                                                                                                        25
 2x
                                         1 + π , − π < x < 0
                                         
12. In the Fourier expansion of f ( x) =                     in (−π , π ) . Find the value of bn ,
                                         1 − 2 x , 0 < x < π
                                          π
                                         
the coefficient of sin nx.
Ans:
             Since f (x) is an even function the value of bn = 0.

                                                  2( − x)  2x         
 In − π ≤ x ≤ 0 i.e., 0 ≤ − x ≤ π , f (− x ) = 1 − π = 1 + π = f ( x )
                                                                      
13. What is the constant term and the coefficient of cos nx, a n in the Fourier expansion of

f ( x) = x − x 3 in (-7 , 7)?
Ans:
Given                 f ( x) = x − x 3

                      f ( − x ) = − x + x 3 = −( x − x 3 ) = − f ( x )

The given function is an odd function. Hence a 0 and a n are zero.


14. State Parseval’s identity for full range expansion of f (x ) as Fourier series in (0 , 2l).
Ans:
             c + 2l                      2                 2            2
        1                              ∞       ∞

               ∫ [ f ( x)] dx. = ao + ∑ a n + ∑ bn .
                           2

        2l     c                  4   n =1 2  n =1 2


       where
                                                  c + 2l
                                              1                         nπx
                                       an =
                                              l     ∫
                                                    c
                                                           f ( x) cos
                                                                         l
                                                                            dx, n ≥ 0

                                                  c + 2l
                                              1                         nπx
                                       bn =
                                              l     ∫
                                                    c
                                                           f ( x) sin
                                                                         l
                                                                            dx, n ≥ 1

15. Find a Fourier sine series for the function f (x ) = 1; 0 < x < π .
Ans:
                                                     ∞
The Fourier sine series of f ( x) = ∑ bn sin nx                                         …………………….(1)
                                                    n =1




                                                                                                   26
π
                                    2
                            bn =
                                    π   ∫ f ( x) sin nx dx
                                        0
                                        π                        π
                                              2  − cos nx 
                                2
                               = ∫ sin nx dx = 
                                π 0           π  n 0
                                                            =−
                                                                 2
                                                                nπ
                                                                   ( (−1) n − 1)
                            bn = 0, when ' n' is even
                                   4
                                =     , when ' n' is odd
                                  nπ
                                           ∞
                                                    4
                            ∴ f ( x) = ∑              . sin nπ
                                       n =1, 3, 5, nπ


                                                     0     0< x<π
16. If the Fourier series for the function f ( x ) =                  is
                                                     sin x 0 < x < 2π

           − 1 2  cos 2 x cos 4 x     1                    1   1   1         π −2
f ( x) =      +          +        +  + sin x Deduce that    −   +    − ∞ =      .
           π π  1.3        3.5        2                   1.3 3.5 5.7          4
Ans:
                          π
            Putting x =     we get
                          2
                      π  −1 2  1   1   1        1
                    f =    + −   +   −    +  ∞ +
                      2  π π  1.3 3.5 5.7       2
                            −1 2  1   1   1       1
                       0=     + −   +   −    + ∞ +
                            π π  1.3 3.5 5.7      2
 1   1   1         π −2
   −   +    − ∞ =      .
1.3 3.5 5.7          4




17. Define Root mean square value of a function?
Ans:
                                                                          c +2 l
                                                                     1
                                                                            ∫y
                                                                                   2
           If a function y = f (x ) is defined in (c , c+2l), then                     dx is called the root mean-
                                                                     2l     c


square(R.M.S.) value of y in (c , c+2l) and is denoted by y.



                                                                                                                 27
c + 2l
                       2 1
                Thus y =        ∫y
                                       2
                                           dx.
                         2l     c


18. If f ( x) = x 2 + x is expressed as a Fourier series in the interval (-2 , 2), to which value this
series converges at x = 2.
Ans:
        Since x = 2 is a point of continuity, the Fourier series converges to the arithmetic mean of
f (x) at x = -2 and x = 2
                  f (2) + f (−2) 4 − 2 + 4 + 2
        i.e.,                   =              =4
                         2             2
19. If the Fourier series corresponding to f ( x ) = x in the interval (0 , 2π ) is

a0 ∞
  + ∑ (a n cos nx + bn sin nx), without finding the values of a 0, a n , bn find the value of
2 n =1

   2  ∞
a0
   + ∑ (a n + bn ).
           2    2

 2   n =1

Ans:
By using Parseval’s identity,
           2                   2π                    2π
         a0    ∞
                             1          1  x3  8
            + ∑ (a n + bn ) = ∫ x 2 dx =   = π 2 .
                    2    2

          2                  π 0           3 
                                        π  0   3
              n =1


20. Find the constant term in the Fourier series corresponding to f ( x) = cos 2 x expressed in the

interval (−π , π ) .
Ans:
       Given f ( x) = cos 2 x
                  π                    π                              π
           1                    1  1 + cos 2 x  1   sin 2 x 
 Now a 0 =        ∫π cos x dx = π −∫π  2 dx = π  x + 2  0 = 1
                       2

           π      −                                         


                                                 PART B
1. (i) Express f ( x) = x sin x as a Fourier series in 0 ≤ x ≤ 2π .




                                                                                                     28
2l  πx 1      2πx 1    3πx 
  (ii) Show that for 0 < x <l, x =                  sin − sin    + sin     . Using root mean square
                                                p      l 2     l  3     l  

                                            1   1   1
value of x, deduce the value of             2
                                              + 2 + 2 +
                                           1   2   3
2. (i) Find the Fourier series of periodicity 3 for f ( x ) = 2 x − x 2 in 0 < x < 3.
  (ii) Find the Fourier series expansion of period 2 π for the function y = f (x) which is defined

in (0 , 2π ) by means of the table of values given below. Find the series upto the third harmonic.
             x               0             π           2π        π         4π         5π        2π
                                           3            3                   3          3
           f (x )            1.0          1.4          1.9       1.7       1.5        1.2       1.0


3.(i) Find the Fourier series of periodicity 2 π for f ( x) = x 2 for 0 < x < 2 π .

                                                l 4l        πx 1     3πx    
   (ii) Show that for 0 < x <l, x =              −       cos + 2 cos     +  . Deduce that
                                                2 π2         l 3      l     

 1   1   1      π4
   + 4 + 4 + =    .
14 3    5       96
                                            l − x, 0 < x ≤ l
4. (i) Find the Fourier series for f ( x) =                   . Hence deduce the sum to infinity of
                                            0,     l ≤ x ≤ 2l

              ∞
                         1
the series   ∑ (2n + 1)
             n =0
                              2
                                  .

   (ii) Find the complex form of Fourier series of f ( x ) = e ax (−π < x < π ) in the form

         sinh aπ    ∞
                                    a + in inx                        π     ∞
                                                                                (−1) n
e ax =
            π
                    ∑ (−1) n
                    −∞             a2 + n2
                                           e and hence prove that          =∑ 2
                                                                  a sinh aπ −∞ n + a 2
                                                                                       .

5. Obtain the half range cosine series for f ( x) = x in (0 , π ).

6. Find the Fourier series for f ( x) = cos x in the interval (−π , π ) .

                                                                                 1   1      π3
7. (i) Expanding x(π − x) as a sine series in (0 , π ) show that 1 −               + 3 + =    .
                                                                                 33 5       32
  (ii) Find the Fourier series as far as the second harmonic to represent the function given in the
following data.
                         x            0            1         2         3          4         5



                                                                                                      29
f (x )       9            18          24          28         26        20


8. Obtain the Fourier series for f (x ) of period 2l and defined as follows
               L + x in ( − L,0)
     f ( x) = 
               L − x in (0, L)
                        1   1   1      π2
Hence deduce that         + 2 + 2 + =    .
                       12 3    5       8
9. Obtain the half range cosine series for f ( x) = x in (0 , π ).
                                             1 in (0, π )
10. (i) Find the Fourier series of f ( x ) = 
                                             2 in (π ,2π )
    (ii) Obtain the sine series for the function
                                         l
                            x in 0 ≤ x ≤ 2
                           
                  f ( x) = 
                           l − x in l ≤ x ≤ l
                           
                                    2
11. (i) Find the Fourier series for the function
                  0 in (−1, 0)
         f ( x) =              and f ( x + 2) = f ( x ) for all x.
                  1 in (0, 1)
    (ii) Determine the Fourier series for the function
                   πx,         0 ≤ x ≤1
          f ( x) = 
                   π (2 − x ), 1 ≤ x ≤ 2
12. Obtain the Fourier series for f ( x ) = 1 + x + x 2 in (−π , π ) . Deduce that

 1   1   1      π2
   + 2 + 2 + =    .
12 2    3       6


13. Obtain the constant term and the first harmonic in the Fourier series expansion for f (x)

where f (x) is given in the following table.
            x         0     1        2       3     4          5    6     7    8     9    10    11
          f (x)      18.   18.      17.     15.   11.        8.3   6.    5.   6.    9.   12.   15.7
                                                                   0     3    4     0    4
                      0    7        6        0     6


14. (i) Express f ( x) = x sin x as a Fourier series in (−π , π ).


                                                                                                      30
(ii) Obtain the half range cosine series for f ( x) = ( x − 2) 2 in the interval 0 < x < 2.
15. Find the half range sine series of f ( x) = x cos x in (0 , π ).
16. (i) Find the Fourier series expansion of f (x ) = e − x in (−π , π )
    (ii) Find the half-range sine series of f (x ) = sin ax in (0 , l).
17. Expand f (x ) = x - x 2 as a Fourier series in -1 < x < 1 and using this series find the r.m.s.

value of f (x) in the interval.
18. The following table gives the variations of a periodic function over a period T.
                              0         T            T           T   2T                5T          T
                 x                      6             3          2    3                 6
               f (x)       1.98         1.3         1.05        1.3 -0.88             -0.25       1.98
                                                                    2πx
      Show that f (x ) = 0.75 + 0.37 cos θ +1.004 sin θ , where θ =
                                                                     T
19. Find the Fourier series upto the third harmonic for the function y = f (x) defined in (0 , π )
from the table
                  x           0          π          2π          3π          4π         5π          π
                                         6           6           6           6          6
                 f (x)      2.34        2.2         1.6        0.83        0.51       0.88        1.19


20. (i) Find the half-range (i) cosine series and (ii) sine series for f (x ) = x 2 in (0 , π )
    (ii) Find the complex form of the Fourier series of f (x) = cos ax in (- π , π ).




                                                                                                         31

Más contenido relacionado

La actualidad más candente

03 truncation errors
03 truncation errors03 truncation errors
03 truncation errorsmaheej
 
Half range sine and cosine series
Half range sine and cosine seriesHalf range sine and cosine series
Half range sine and cosine seriesChandan S
 
Laplace equation
Laplace equationLaplace equation
Laplace equationalexkhan129
 
Numerical integration;Gaussian integration one point, two point and three poi...
Numerical integration;Gaussian integration one point, two point and three poi...Numerical integration;Gaussian integration one point, two point and three poi...
Numerical integration;Gaussian integration one point, two point and three poi...vaibhav tailor
 
Eigenvalues and Eigenvectors
Eigenvalues and EigenvectorsEigenvalues and Eigenvectors
Eigenvalues and EigenvectorsVinod Srivastava
 
Newton's Backward Interpolation Formula with Example
Newton's Backward Interpolation Formula with ExampleNewton's Backward Interpolation Formula with Example
Newton's Backward Interpolation Formula with ExampleMuhammadUsmanIkram2
 
Methods of variation of parameters- advance engineering mathe mathematics
Methods of variation of parameters- advance engineering mathe mathematicsMethods of variation of parameters- advance engineering mathe mathematics
Methods of variation of parameters- advance engineering mathe mathematicsKaushal Patel
 
Fast Fourier Transform
Fast Fourier TransformFast Fourier Transform
Fast Fourier Transformop205
 
aem : Fourier series of Even and Odd Function
aem :  Fourier series of Even and Odd Functionaem :  Fourier series of Even and Odd Function
aem : Fourier series of Even and Odd FunctionSukhvinder Singh
 
Engineering Numerical Analysis Lecture-1
Engineering Numerical Analysis Lecture-1Engineering Numerical Analysis Lecture-1
Engineering Numerical Analysis Lecture-1Muhammad Waqas
 
Vector calculus
Vector calculusVector calculus
Vector calculusraghu ram
 
Integral Transform
Integral  TransformIntegral  Transform
Integral TransformSheharBano31
 

La actualidad más candente (20)

03 truncation errors
03 truncation errors03 truncation errors
03 truncation errors
 
Half range sine and cosine series
Half range sine and cosine seriesHalf range sine and cosine series
Half range sine and cosine series
 
Laplace equation
Laplace equationLaplace equation
Laplace equation
 
Newton Raphson Method
Newton Raphson MethodNewton Raphson Method
Newton Raphson Method
 
Numerical integration;Gaussian integration one point, two point and three poi...
Numerical integration;Gaussian integration one point, two point and three poi...Numerical integration;Gaussian integration one point, two point and three poi...
Numerical integration;Gaussian integration one point, two point and three poi...
 
Chapter 4 (maths 3)
Chapter 4 (maths 3)Chapter 4 (maths 3)
Chapter 4 (maths 3)
 
Fourier series
Fourier seriesFourier series
Fourier series
 
Eigenvalues and Eigenvectors
Eigenvalues and EigenvectorsEigenvalues and Eigenvectors
Eigenvalues and Eigenvectors
 
Newton's Backward Interpolation Formula with Example
Newton's Backward Interpolation Formula with ExampleNewton's Backward Interpolation Formula with Example
Newton's Backward Interpolation Formula with Example
 
Methods of variation of parameters- advance engineering mathe mathematics
Methods of variation of parameters- advance engineering mathe mathematicsMethods of variation of parameters- advance engineering mathe mathematics
Methods of variation of parameters- advance engineering mathe mathematics
 
Inverse laplace transforms
Inverse laplace transformsInverse laplace transforms
Inverse laplace transforms
 
Fourier transforms
Fourier transforms Fourier transforms
Fourier transforms
 
Laplace transforms
Laplace transformsLaplace transforms
Laplace transforms
 
Fast Fourier Transform
Fast Fourier TransformFast Fourier Transform
Fast Fourier Transform
 
aem : Fourier series of Even and Odd Function
aem :  Fourier series of Even and Odd Functionaem :  Fourier series of Even and Odd Function
aem : Fourier series of Even and Odd Function
 
Engineering Numerical Analysis Lecture-1
Engineering Numerical Analysis Lecture-1Engineering Numerical Analysis Lecture-1
Engineering Numerical Analysis Lecture-1
 
1 d heat equation
1 d heat equation1 d heat equation
1 d heat equation
 
Vector calculus
Vector calculusVector calculus
Vector calculus
 
Bisection method
Bisection methodBisection method
Bisection method
 
Integral Transform
Integral  TransformIntegral  Transform
Integral Transform
 

Destacado

Numerical Solution of Nth - Order Fuzzy Initial Value Problems by Fourth Orde...
Numerical Solution of Nth - Order Fuzzy Initial Value Problems by Fourth Orde...Numerical Solution of Nth - Order Fuzzy Initial Value Problems by Fourth Orde...
Numerical Solution of Nth - Order Fuzzy Initial Value Problems by Fourth Orde...IOSR Journals
 
N.P. BALI ENGINEERING MATH REAL
N.P. BALI ENGINEERING MATH REALN.P. BALI ENGINEERING MATH REAL
N.P. BALI ENGINEERING MATH REALchucky oz
 
Free Ebooks Download ! Edhole
Free Ebooks Download ! EdholeFree Ebooks Download ! Edhole
Free Ebooks Download ! EdholeEdhole.com
 
Free Download Powerpoint Slides
Free Download Powerpoint SlidesFree Download Powerpoint Slides
Free Download Powerpoint SlidesGeorge
 

Destacado (8)

Chapter 1 (maths 3)
Chapter 1 (maths 3)Chapter 1 (maths 3)
Chapter 1 (maths 3)
 
Chapter 3 (maths 3)
Chapter 3 (maths 3)Chapter 3 (maths 3)
Chapter 3 (maths 3)
 
Chapter 5 (maths 3)
Chapter 5 (maths 3)Chapter 5 (maths 3)
Chapter 5 (maths 3)
 
mathematics formulas
mathematics formulasmathematics formulas
mathematics formulas
 
Numerical Solution of Nth - Order Fuzzy Initial Value Problems by Fourth Orde...
Numerical Solution of Nth - Order Fuzzy Initial Value Problems by Fourth Orde...Numerical Solution of Nth - Order Fuzzy Initial Value Problems by Fourth Orde...
Numerical Solution of Nth - Order Fuzzy Initial Value Problems by Fourth Orde...
 
N.P. BALI ENGINEERING MATH REAL
N.P. BALI ENGINEERING MATH REALN.P. BALI ENGINEERING MATH REAL
N.P. BALI ENGINEERING MATH REAL
 
Free Ebooks Download ! Edhole
Free Ebooks Download ! EdholeFree Ebooks Download ! Edhole
Free Ebooks Download ! Edhole
 
Free Download Powerpoint Slides
Free Download Powerpoint SlidesFree Download Powerpoint Slides
Free Download Powerpoint Slides
 

Similar a Fourier Series Coefficients

Varian, microeconomic analysis, solution book
Varian, microeconomic analysis, solution bookVarian, microeconomic analysis, solution book
Varian, microeconomic analysis, solution bookJosé Antonio PAYANO YALE
 
Mathematical physics group 16
Mathematical physics group 16Mathematical physics group 16
Mathematical physics group 16derry92
 
IJCER (www.ijceronline.com) International Journal of computational Engineerin...
IJCER (www.ijceronline.com) International Journal of computational Engineerin...IJCER (www.ijceronline.com) International Journal of computational Engineerin...
IJCER (www.ijceronline.com) International Journal of computational Engineerin...ijceronline
 
Fourier series of odd functions with period 2 l
Fourier series of odd functions with period 2 lFourier series of odd functions with period 2 l
Fourier series of odd functions with period 2 lPepa Vidosa Serradilla
 
Lattices and codes
Lattices and codesLattices and codes
Lattices and codesSpringer
 
Math 1102-ch-3-lecture note Fourier Series.pdf
Math 1102-ch-3-lecture note Fourier Series.pdfMath 1102-ch-3-lecture note Fourier Series.pdf
Math 1102-ch-3-lecture note Fourier Series.pdfhabtamu292245
 
Cs229 cvxopt
Cs229 cvxoptCs229 cvxopt
Cs229 cvxoptcerezaso
 
Fourier 3
Fourier 3Fourier 3
Fourier 3nugon
 
Welcome to International Journal of Engineering Research and Development (IJERD)
Welcome to International Journal of Engineering Research and Development (IJERD)Welcome to International Journal of Engineering Research and Development (IJERD)
Welcome to International Journal of Engineering Research and Development (IJERD)IJERD Editor
 
Welcome to International Journal of Engineering Research and Development (IJERD)
Welcome to International Journal of Engineering Research and Development (IJERD)Welcome to International Journal of Engineering Research and Development (IJERD)
Welcome to International Journal of Engineering Research and Development (IJERD)IJERD Editor
 
02 2d systems matrix
02 2d systems matrix02 2d systems matrix
02 2d systems matrixRumah Belajar
 
Lesson 14: Derivatives of Logarithmic and Exponential Functions (slides)
Lesson 14: Derivatives of Logarithmic and Exponential Functions (slides)Lesson 14: Derivatives of Logarithmic and Exponential Functions (slides)
Lesson 14: Derivatives of Logarithmic and Exponential Functions (slides)Matthew Leingang
 
Lesson 14: Derivatives of Logarithmic and Exponential Functions (slides)
Lesson 14: Derivatives of Logarithmic and Exponential Functions (slides)Lesson 14: Derivatives of Logarithmic and Exponential Functions (slides)
Lesson 14: Derivatives of Logarithmic and Exponential Functions (slides)Mel Anthony Pepito
 
SOME THOUGHTS ON DIVERGENT SERIES
SOME THOUGHTS ON DIVERGENT SERIESSOME THOUGHTS ON DIVERGENT SERIES
SOME THOUGHTS ON DIVERGENT SERIESgenius98
 

Similar a Fourier Series Coefficients (20)

Fourier series
Fourier seriesFourier series
Fourier series
 
Fourier series 2
Fourier series 2Fourier series 2
Fourier series 2
 
Fourier series
Fourier seriesFourier series
Fourier series
 
Fourier series
Fourier seriesFourier series
Fourier series
 
Varian, microeconomic analysis, solution book
Varian, microeconomic analysis, solution bookVarian, microeconomic analysis, solution book
Varian, microeconomic analysis, solution book
 
Mathematical physics group 16
Mathematical physics group 16Mathematical physics group 16
Mathematical physics group 16
 
IJCER (www.ijceronline.com) International Journal of computational Engineerin...
IJCER (www.ijceronline.com) International Journal of computational Engineerin...IJCER (www.ijceronline.com) International Journal of computational Engineerin...
IJCER (www.ijceronline.com) International Journal of computational Engineerin...
 
Fourier series of odd functions with period 2 l
Fourier series of odd functions with period 2 lFourier series of odd functions with period 2 l
Fourier series of odd functions with period 2 l
 
Lattices and codes
Lattices and codesLattices and codes
Lattices and codes
 
Cap4 lec5
Cap4 lec5Cap4 lec5
Cap4 lec5
 
Math 1102-ch-3-lecture note Fourier Series.pdf
Math 1102-ch-3-lecture note Fourier Series.pdfMath 1102-ch-3-lecture note Fourier Series.pdf
Math 1102-ch-3-lecture note Fourier Series.pdf
 
Cs229 cvxopt
Cs229 cvxoptCs229 cvxopt
Cs229 cvxopt
 
Fourier 3
Fourier 3Fourier 3
Fourier 3
 
Fourier series 1
Fourier series 1Fourier series 1
Fourier series 1
 
Welcome to International Journal of Engineering Research and Development (IJERD)
Welcome to International Journal of Engineering Research and Development (IJERD)Welcome to International Journal of Engineering Research and Development (IJERD)
Welcome to International Journal of Engineering Research and Development (IJERD)
 
Welcome to International Journal of Engineering Research and Development (IJERD)
Welcome to International Journal of Engineering Research and Development (IJERD)Welcome to International Journal of Engineering Research and Development (IJERD)
Welcome to International Journal of Engineering Research and Development (IJERD)
 
02 2d systems matrix
02 2d systems matrix02 2d systems matrix
02 2d systems matrix
 
Lesson 14: Derivatives of Logarithmic and Exponential Functions (slides)
Lesson 14: Derivatives of Logarithmic and Exponential Functions (slides)Lesson 14: Derivatives of Logarithmic and Exponential Functions (slides)
Lesson 14: Derivatives of Logarithmic and Exponential Functions (slides)
 
Lesson 14: Derivatives of Logarithmic and Exponential Functions (slides)
Lesson 14: Derivatives of Logarithmic and Exponential Functions (slides)Lesson 14: Derivatives of Logarithmic and Exponential Functions (slides)
Lesson 14: Derivatives of Logarithmic and Exponential Functions (slides)
 
SOME THOUGHTS ON DIVERGENT SERIES
SOME THOUGHTS ON DIVERGENT SERIESSOME THOUGHTS ON DIVERGENT SERIES
SOME THOUGHTS ON DIVERGENT SERIES
 

Último

Tampa BSides - Chef's Tour of Microsoft Security Adoption Framework (SAF)
Tampa BSides - Chef's Tour of Microsoft Security Adoption Framework (SAF)Tampa BSides - Chef's Tour of Microsoft Security Adoption Framework (SAF)
Tampa BSides - Chef's Tour of Microsoft Security Adoption Framework (SAF)Mark Simos
 
unit 4 immunoblotting technique complete.pptx
unit 4 immunoblotting technique complete.pptxunit 4 immunoblotting technique complete.pptx
unit 4 immunoblotting technique complete.pptxBkGupta21
 
Take control of your SAP testing with UiPath Test Suite
Take control of your SAP testing with UiPath Test SuiteTake control of your SAP testing with UiPath Test Suite
Take control of your SAP testing with UiPath Test SuiteDianaGray10
 
The Role of FIDO in a Cyber Secure Netherlands: FIDO Paris Seminar.pptx
The Role of FIDO in a Cyber Secure Netherlands: FIDO Paris Seminar.pptxThe Role of FIDO in a Cyber Secure Netherlands: FIDO Paris Seminar.pptx
The Role of FIDO in a Cyber Secure Netherlands: FIDO Paris Seminar.pptxLoriGlavin3
 
DevEX - reference for building teams, processes, and platforms
DevEX - reference for building teams, processes, and platformsDevEX - reference for building teams, processes, and platforms
DevEX - reference for building teams, processes, and platformsSergiu Bodiu
 
Moving Beyond Passwords: FIDO Paris Seminar.pdf
Moving Beyond Passwords: FIDO Paris Seminar.pdfMoving Beyond Passwords: FIDO Paris Seminar.pdf
Moving Beyond Passwords: FIDO Paris Seminar.pdfLoriGlavin3
 
Generative AI for Technical Writer or Information Developers
Generative AI for Technical Writer or Information DevelopersGenerative AI for Technical Writer or Information Developers
Generative AI for Technical Writer or Information DevelopersRaghuram Pandurangan
 
Dev Dives: Streamline document processing with UiPath Studio Web
Dev Dives: Streamline document processing with UiPath Studio WebDev Dives: Streamline document processing with UiPath Studio Web
Dev Dives: Streamline document processing with UiPath Studio WebUiPathCommunity
 
Nell’iperspazio con Rocket: il Framework Web di Rust!
Nell’iperspazio con Rocket: il Framework Web di Rust!Nell’iperspazio con Rocket: il Framework Web di Rust!
Nell’iperspazio con Rocket: il Framework Web di Rust!Commit University
 
Scanning the Internet for External Cloud Exposures via SSL Certs
Scanning the Internet for External Cloud Exposures via SSL CertsScanning the Internet for External Cloud Exposures via SSL Certs
Scanning the Internet for External Cloud Exposures via SSL CertsRizwan Syed
 
Are Multi-Cloud and Serverless Good or Bad?
Are Multi-Cloud and Serverless Good or Bad?Are Multi-Cloud and Serverless Good or Bad?
Are Multi-Cloud and Serverless Good or Bad?Mattias Andersson
 
Transcript: New from BookNet Canada for 2024: Loan Stars - Tech Forum 2024
Transcript: New from BookNet Canada for 2024: Loan Stars - Tech Forum 2024Transcript: New from BookNet Canada for 2024: Loan Stars - Tech Forum 2024
Transcript: New from BookNet Canada for 2024: Loan Stars - Tech Forum 2024BookNet Canada
 
"Debugging python applications inside k8s environment", Andrii Soldatenko
"Debugging python applications inside k8s environment", Andrii Soldatenko"Debugging python applications inside k8s environment", Andrii Soldatenko
"Debugging python applications inside k8s environment", Andrii SoldatenkoFwdays
 
WordPress Websites for Engineers: Elevate Your Brand
WordPress Websites for Engineers: Elevate Your BrandWordPress Websites for Engineers: Elevate Your Brand
WordPress Websites for Engineers: Elevate Your Brandgvaughan
 
Streamlining Python Development: A Guide to a Modern Project Setup
Streamlining Python Development: A Guide to a Modern Project SetupStreamlining Python Development: A Guide to a Modern Project Setup
Streamlining Python Development: A Guide to a Modern Project SetupFlorian Wilhelm
 
A Deep Dive on Passkeys: FIDO Paris Seminar.pptx
A Deep Dive on Passkeys: FIDO Paris Seminar.pptxA Deep Dive on Passkeys: FIDO Paris Seminar.pptx
A Deep Dive on Passkeys: FIDO Paris Seminar.pptxLoriGlavin3
 
The Fit for Passkeys for Employee and Consumer Sign-ins: FIDO Paris Seminar.pptx
The Fit for Passkeys for Employee and Consumer Sign-ins: FIDO Paris Seminar.pptxThe Fit for Passkeys for Employee and Consumer Sign-ins: FIDO Paris Seminar.pptx
The Fit for Passkeys for Employee and Consumer Sign-ins: FIDO Paris Seminar.pptxLoriGlavin3
 
Transcript: New from BookNet Canada for 2024: BNC CataList - Tech Forum 2024
Transcript: New from BookNet Canada for 2024: BNC CataList - Tech Forum 2024Transcript: New from BookNet Canada for 2024: BNC CataList - Tech Forum 2024
Transcript: New from BookNet Canada for 2024: BNC CataList - Tech Forum 2024BookNet Canada
 
Merck Moving Beyond Passwords: FIDO Paris Seminar.pptx
Merck Moving Beyond Passwords: FIDO Paris Seminar.pptxMerck Moving Beyond Passwords: FIDO Paris Seminar.pptx
Merck Moving Beyond Passwords: FIDO Paris Seminar.pptxLoriGlavin3
 

Último (20)

Tampa BSides - Chef's Tour of Microsoft Security Adoption Framework (SAF)
Tampa BSides - Chef's Tour of Microsoft Security Adoption Framework (SAF)Tampa BSides - Chef's Tour of Microsoft Security Adoption Framework (SAF)
Tampa BSides - Chef's Tour of Microsoft Security Adoption Framework (SAF)
 
unit 4 immunoblotting technique complete.pptx
unit 4 immunoblotting technique complete.pptxunit 4 immunoblotting technique complete.pptx
unit 4 immunoblotting technique complete.pptx
 
Take control of your SAP testing with UiPath Test Suite
Take control of your SAP testing with UiPath Test SuiteTake control of your SAP testing with UiPath Test Suite
Take control of your SAP testing with UiPath Test Suite
 
The Role of FIDO in a Cyber Secure Netherlands: FIDO Paris Seminar.pptx
The Role of FIDO in a Cyber Secure Netherlands: FIDO Paris Seminar.pptxThe Role of FIDO in a Cyber Secure Netherlands: FIDO Paris Seminar.pptx
The Role of FIDO in a Cyber Secure Netherlands: FIDO Paris Seminar.pptx
 
DevEX - reference for building teams, processes, and platforms
DevEX - reference for building teams, processes, and platformsDevEX - reference for building teams, processes, and platforms
DevEX - reference for building teams, processes, and platforms
 
Moving Beyond Passwords: FIDO Paris Seminar.pdf
Moving Beyond Passwords: FIDO Paris Seminar.pdfMoving Beyond Passwords: FIDO Paris Seminar.pdf
Moving Beyond Passwords: FIDO Paris Seminar.pdf
 
Generative AI for Technical Writer or Information Developers
Generative AI for Technical Writer or Information DevelopersGenerative AI for Technical Writer or Information Developers
Generative AI for Technical Writer or Information Developers
 
Dev Dives: Streamline document processing with UiPath Studio Web
Dev Dives: Streamline document processing with UiPath Studio WebDev Dives: Streamline document processing with UiPath Studio Web
Dev Dives: Streamline document processing with UiPath Studio Web
 
Nell’iperspazio con Rocket: il Framework Web di Rust!
Nell’iperspazio con Rocket: il Framework Web di Rust!Nell’iperspazio con Rocket: il Framework Web di Rust!
Nell’iperspazio con Rocket: il Framework Web di Rust!
 
Scanning the Internet for External Cloud Exposures via SSL Certs
Scanning the Internet for External Cloud Exposures via SSL CertsScanning the Internet for External Cloud Exposures via SSL Certs
Scanning the Internet for External Cloud Exposures via SSL Certs
 
DMCC Future of Trade Web3 - Special Edition
DMCC Future of Trade Web3 - Special EditionDMCC Future of Trade Web3 - Special Edition
DMCC Future of Trade Web3 - Special Edition
 
Are Multi-Cloud and Serverless Good or Bad?
Are Multi-Cloud and Serverless Good or Bad?Are Multi-Cloud and Serverless Good or Bad?
Are Multi-Cloud and Serverless Good or Bad?
 
Transcript: New from BookNet Canada for 2024: Loan Stars - Tech Forum 2024
Transcript: New from BookNet Canada for 2024: Loan Stars - Tech Forum 2024Transcript: New from BookNet Canada for 2024: Loan Stars - Tech Forum 2024
Transcript: New from BookNet Canada for 2024: Loan Stars - Tech Forum 2024
 
"Debugging python applications inside k8s environment", Andrii Soldatenko
"Debugging python applications inside k8s environment", Andrii Soldatenko"Debugging python applications inside k8s environment", Andrii Soldatenko
"Debugging python applications inside k8s environment", Andrii Soldatenko
 
WordPress Websites for Engineers: Elevate Your Brand
WordPress Websites for Engineers: Elevate Your BrandWordPress Websites for Engineers: Elevate Your Brand
WordPress Websites for Engineers: Elevate Your Brand
 
Streamlining Python Development: A Guide to a Modern Project Setup
Streamlining Python Development: A Guide to a Modern Project SetupStreamlining Python Development: A Guide to a Modern Project Setup
Streamlining Python Development: A Guide to a Modern Project Setup
 
A Deep Dive on Passkeys: FIDO Paris Seminar.pptx
A Deep Dive on Passkeys: FIDO Paris Seminar.pptxA Deep Dive on Passkeys: FIDO Paris Seminar.pptx
A Deep Dive on Passkeys: FIDO Paris Seminar.pptx
 
The Fit for Passkeys for Employee and Consumer Sign-ins: FIDO Paris Seminar.pptx
The Fit for Passkeys for Employee and Consumer Sign-ins: FIDO Paris Seminar.pptxThe Fit for Passkeys for Employee and Consumer Sign-ins: FIDO Paris Seminar.pptx
The Fit for Passkeys for Employee and Consumer Sign-ins: FIDO Paris Seminar.pptx
 
Transcript: New from BookNet Canada for 2024: BNC CataList - Tech Forum 2024
Transcript: New from BookNet Canada for 2024: BNC CataList - Tech Forum 2024Transcript: New from BookNet Canada for 2024: BNC CataList - Tech Forum 2024
Transcript: New from BookNet Canada for 2024: BNC CataList - Tech Forum 2024
 
Merck Moving Beyond Passwords: FIDO Paris Seminar.pptx
Merck Moving Beyond Passwords: FIDO Paris Seminar.pptxMerck Moving Beyond Passwords: FIDO Paris Seminar.pptx
Merck Moving Beyond Passwords: FIDO Paris Seminar.pptx
 

Fourier Series Coefficients

  • 1. CHAPTER 2 FOURIER SERIES PERIODIC FUNCTIONS A function f (x ) is said to have a period T if for all x, f ( x + T ) = f ( x) , where T is a positive constant. The least value of T>0 is called the period of f (x) . EXAMPLES We know that f (x ) = sin x = sin (x + 4 π ) = … Therefore the function has period 2 π , 4 π , 6 π , etc. However, 2 is the least value and therefore is the period of f(x). Similarly cos x is a periodic function with the period 2 π and tan x has period π . DIRICHLET’S CONDITIONS A function f (x ) defined in c ≤ x ≤ c+2l can be expanded as an infinite trigonometric a nπx nπx series of the form o + ∑ a n cos + ∑ bn sin , provided 2 l l 1. f (x) is single- valued and finite in (c , c+2l) 2. f (x) is continuous or piecewise continuous with finite number of finite discontinuities in (c , c+2l). 3. f (x) has no or finite number of maxima or minima in (c , c+2l). EULER’S FORMULAS If a function f (x) defined in (c , c+2l) can be expanded as the infinite trigonometric ao ∞ nπx ∞ nπx series 2 + ∑ an cos n =1 l + ∑ bn sin n =1 l then c + 2l 1 nπx an = l ∫ c f ( x) cos l dx, n ≥ 0 c + 2l 1 nπx bn = l ∫c f ( x) sin l dx, n ≥ 1 [ Formulas given above for a n and bn are called Euler’s formulas for Fourier coefficients]
  • 2. DEFINITION OF FOURIER SERIES ao ∞ nπx ∞ nπx The infinite trigonometric series 2 + ∑ an cos n =1 l + ∑ bn sin n =1 l is called the Fourier series of f (x) in the interval c ≤ x ≤ c+2l, provided the coefficients are given by the Euler’s formulas. EVEN FUNCTION If f (x) = φ (x) in (-l , l) such that φ (− x) = φ (x) , then f (x ) is said to be an even function of x in (-l , l). φ1 ( x ) in (−l ,0) If f ( x) =  φ 2 ( x) in (0, l ) Such that φ1 (− x) = φ 2 ( x) or φ 2 ( − x ) = φ1 ( x ) , then f (x) is said to be an even function of x in (-l , l). EXAMPLE y = cos x , y = x 2 are even functions. ODD FUNCTION If f (x) = φ (x) in (-l , l) such that φ (− x) = - φ (x) , then f (x) is said to be an odd function of x in (-l , l). φ1 ( x ) in (−l ,0) If f ( x) =  φ 2 ( x) in (0, l ) Such that φ1 (− x) = - φ 2 ( x ) or φ 2 ( − x ) = - φ1 ( x ) , then f (x) is said to be an odd function of x in (-l , l). EXAMPLE y = sin x , y = x are odd functions. FOURIER SERIES OF EVEN AND ODD FUNCTIONS 1. The Fourier series of an even function f (x ) in (-l , l) contains only cosine terms 2
  • 3. (constant term included), i.e. the Fourier series of an even function f (x) in (-l , l) is given by ao nπx f (x) = 2 + ∑a n cos l , nπx l 2 where a n = ∫ f ( x ) cos dx. l 0 l 2. The Fourier series of an odd function f (x) in (-l , l) contains only sine terms, i.e. the Fourier series of an odd function f (x ) in (-l , l) is given by nπx f (x) = ∑b n sin l , nπx l 2 where bn = ∫ f ( x) sin l dx. l 0 PROBLEMS 1. Find the Fourier series of period 2l for the function f (x ) = x(2l – x) in (0 , 2l). Deduce 1 1 1 the sum of f (x) = 2 − 2 + 2 − 1 2 3 Solution: ao ∞ nπx ∞ nπx Let f (x ) = 2 + ∑a n =1 n cos l + ∑ bn sin n =1 l in (0 , 2l) …………(1) nπx 2l 1 an = l0∫ x(2l − x) cos l dx 2l   nπx   nπx   nπx    sin   − cos   − sin  1 2  l  − (2l − 2 x ) l  + (−2) l  , = (2lx − x ) l  nπ   n 2π 2   n 3π 3           l   l2   l3  0 using Bernoulli’s formula. 2 1 = [ − 2l cos 2nπ − 2l ] = − 4l 2 n 2π 2 n 2π 2l 1 x3  2l 1 4 a o = ∫ x(2l − x )dx = lx 2 −  = l 2 . l0 l 3 0 3 3
  • 4. nπx 2l 1 bn = l0∫ x(2l − x) sin l dx =0 Using these values in (1), we have 2 2 4l 2 ∞ 1 nπx x (2l - x) = l − 2 3 π ∑n n =1 2 cos l in (0, 2l) ……………..(2) 1 1 1 The required series 2 − 2 + 2 − … ∞ can be obtained by putting x = l in the Fourier 1 2 3 series in (2). x = l lies in (0 , 2l) and is a point of continuity of the function f (x) = x(2l – x). ∴ [ Sum the Fourier series in (2) ] x =1 = f(l) ∞ 2 2 4l 2 1 i.e. l − 2 3 π ∑n n =1 2 cos nπ = l(2l - l) 4l 2  1 1 1  l 2 i.e.. -  − 2 + 2 − 2 + ...∞  = π2  1 2 3  3 ∴ 1 1 1 π2 − 2 + 2 − …∞ = 12 2 3 12 2. Find the Fourier series of period 2 π for the function f (x) = x cos x in 0 < x < 2 π . Solution: ∞ ∞ ao Let f (x ) = 2 + ∑ an cos nx + ∑ bn sin nx n =1 n =1 .……..…………(1) 2π 1 an = π ∫ x cos x cos nxdx 0 2π 1 = 2π ∫ x[ cos(n + 1) x + cos(n − 1) x]dx 0 1  sin( n + 1) x cos(n + 1) x  2π  sin( n − 1) x cos(n − 1) x  2π  =  x. +  +  x. +  , 2π  n +1 (n + 1) 2  0  n −1 ( n − 1) 2  0    if n ≠ 1 =0, if n ≠ 1 ao = 0 4
  • 5. 2π 1 1 an = ∫ x cos xdx = 2π ∫ x(1 + cos 2 x)dx 2 π 0 0 2π 1 x sin 2 x cos 2 x  2 =  +x +  = π. 2π 2 2 4 0 2π 1 bn = π ∫ x cos x sin nxdx 0 2π 1 = 2π ∫ x[ sin(n + 1) x + sin(n − 1) x]dx 0 1  − cos(n + 1) x sin( n + 1) x  2π  − cos(n − 1) x sin( n − 1) x  2π  =  x. +  +  x. +  , 2π  n +1 (n + 1) 2  0  n −1 (n − 1) 2  0    if n ≠ 1 1 1  1 1  2n =− − = − + =− 2 , if n ≠ 1 n +1 n −1  n + 1 n − 1 n −1 2π 2π 1 1 b1 = π ∫ x cos x sin xdx = 0 2π ∫ x sin 2 xdx 0 2π 1   − cos 2 x  sin 2 x  1 =  x +  =−2 2π   2  4 0 Using these values in (1), we get ∞ 1 n f(x) = π cos x − sin x − 2 ∑ sin nx n = 2 , 3,... n − 1 2 2 3. Find the Fourier series expansion of f (x) = sin ax in (-l , l). Solution: Since f (x) is defined in a range of length 2l, we can expand f (x ) in Fourier series of period 2l. Also f ( − x) = sin[a(-x)] = -sin ax = - f (x) ∴ f (x) is an odd function of x in (-l , l). Hence Fourier series of f (x ) will not contain cosine terms. ∞ nπx Let f (x ) = ∑b n =1 n sin l ………………….(1) 5
  • 6. 1   nπ  nπ l    = ∫ cos l − a  − cos l + a  xdx l 0      l   nπ   nπ   1  sin  l − a  x sin  l + a  x  =    −    l nπ nπ −a +a   l l   0 1  nπ  1  nπ  = sin  − a l − sin  + a l nπ − la  l  nπ + la  l  = 1 nπ − al {− (−1) n sin al} − nπ 1 al {(−1) n sin al} +  1 1  = (−1) n +1 sin al  +   nπ − al nπ + al  (−1) n +1 2nπ sin al = n 2π 2 − a 2 l 2 Using these values in (1), we get ∞ (−1) n +1 n nπx sin ax = 2π sin al ∑ sin n =1 n π −a l 2 2 2 2 l 4. Find the Fourier series expansion of f (x) = e − x in (−π , π ) . Hence obtain a series for cosec π Solution: Though the range (−π , π ) is symmetric about the origin, e − x is neither an even function nor an odd function. ∞ ∞ ao ∴ Let f (x) = 2 + ∑ an cos nx + ∑ bn sin nx n =1 n =1 ..…..…………(1) in (−π , π ) [ the length of the range is 2π ] 6
  • 7. π 1 ∫π e cos nxdx −x an = π− π 1  e −x  =  2 ( − cos nx + n sin nx )  π n +1  −π =− 1 {e −π (−1) n − e π (−1) n } π ( n + 1) 2 2( −1) n = sinh π π (n 2 + 1) 2 sinh π ao = π π 1 ∫π e sin nxdx −x bn = π− π 1  e −x  =  2 ( − sin nx − n cos nx )  π n +1  −π =− n {e −π (−1) n − e π (−1) n } π ( n 2 + 1) 2n(−1) n = sinh π π (n 2 + 1) Using these values in (1), we get sinh π 2 sinh π ∞ (−1) n 2 sinh π ∞ (−1) n n e−x = π + π ∑ n 2 + 1 cos nx + π n =1 ∑ n 2 + 1 sin nx n =1 in (−π , π ) [ Sum of the Fourier series of f ( x )] x =0 = f (0), [Since x=0 is a point of continuity of f(x)] sinh π  ∞ (−1) n  i.e., 1 + 2∑ 2 −0  = e =1 π  n =1 n + 1  −1 ∞ (−1) n i.e., π cos ech π = 1 + 2  + 2∑ 2  2  n=2 n + 1 2 ∞ (−1) n i.e., cos ech π = ∑ π n=2 n 2 + 1 7
  • 8. HALF-RANGE FOURIER SERIES AND PARSEVAL’S THEOREM (i) The half range cosine series in (0 , l) is ao ∞ nπx f (x) = 2 + ∑a n =1 n cos l l 2 l∫ where a o = f ( x )dx. 0 nπx l 2 an = ∫ f ( x) cos l dx. l 0 (ii) The half range sine series in (0 , l) is ∞ nπx f (x) = ∑b n =1 n sin l , nπx l 2 where bn = ∫ f ( x) sin l dx. l 0 (iii) The half range cosine series in (0 , π ) is given by ∞ ao f (x) = 2 + ∑a n =1 n cos nx π 2 where a o = ∫ f ( x )dx. π 0 π 2 π∫ an = f ( x ) cos nxdx. 0 (iv) The half range sine series in (0 , π ) is given by ∞ f (x) = ∑b n =1 n sin nx , π 2 where bn = ∫ f ( x) sin nxdx. π 0 8
  • 9. ROOT-MEAN SQUARE VALUE OF A FUNCTION Definition c +2 l 1 ∫y 2 If a function y = f (x ) is defined in (c , c+2l), then dx is called the root mean- 2l c square(R.M.S.) value of y in (c , c+2l) and is denoted by y. c + 2l 1 2 Thus y = ∫y 2 dx. 2l c PARSEVAL’S THEOREM If y = f (x ) can be expanded as a Fourier series of the form ao ∞ nπx ∞ nπx 2 + ∑ an cos n =1 l + ∑ bn sin n =1 l in (c , c+2l), then the root-mean square value y of y = f (x) in (c , c+2l) is given by 1 2 1 ∞ 1 ∞ y = a o + ∑ a n + ∑ bn 2 2 2 4 2 n =1 2 n =1 PROOF ao ∞ nπx ∞ nπx f (x) = 2 + ∑ an cos n =1 l + ∑ bn sin n =1 l in (c , c+2l) ....……………….(1) ∴ By Euler’s formulas for the Fourier coefficients, c + 2l 1 nπx an = l ∫ c f ( x) cos l dx, n ≥ 0 ..…………………(2) c + 2l 1 nπx bn = l ∫ c f ( x) sin l dx, n ≥ 1 …....……………..(3) Now, by definition, c + 2l c + 2l 1 1 ∫ [ f ( x)] 2 ∫ y dx = 2 y = 2 dx 2l c 2l c c + 2l 1 a ∞ nπx ∞ nπx  = ∫ f ( x)  o + ∑ a n cos + ∑ bn sin dx, using (1) 2l c  2 n =1 l n =1 l  ao1 c + 2l  ∞ a 1 c + 2l nπx  ∞ bn 1 c + 2l nπx  =  ∫ f ( x)dx  + ∑ n  ∫ f ( x) cos dx  + ∑  ∫ f ( x) sin dx  4  l c  n =1 2  l c l  n =1 2  l c l  ∞ ∞ ao an bn = .a o + ∑ .a n + ∑ .bn , by using (2) and (3) 4 n =1 2 n =1 2 9
  • 10. ∞2 ∞ 2 2 ao an b = +∑ +∑ n . 4 n =1 2 n =1 2 EXAMPLES 1. Find the half-range (i) cosine series and (ii) sine series for f (x ) = x 2 in (0 , π ) Solution: (i) To get the half-range cosine series for f (x ) in (0 , π ), we should give an even extension for f (x) in ( − π , 0). i.e. put f (x) = ( − x ) 2 = x 2 in ( − π , 0) Now f (x) is even in ( − π , π ). ∞ ao ∴ f (x) = 2 + ∑a n =1 n cos nx ………………….(1) π 2 an = π ∫ f ( x) cos nxdx. 0 π 2 2 π∫ = x cos nxdx 0 π 2   sin nx   − cos nx   − sin nx  = x2   − 2 x  + 2  π  n   n 2   n 3  0 4 4(−1) n = .π (−1) n = ,n ≠ 0 πn 2 n2 π π 2 2 2 2 2 ao = ∫ f ( x)dx = π ∫ x dx = 3 π π 0 0 ∴ The Fourier half-range cosine series of x 2 is given by π2 ∞ (−1) n x2 = + 4∑ 2 cos nx in (0 , π ). 3 n =1 n (ii) To get the half-range sine series of f (x ) in (0 , π ), we should give an odd extension for f (x) in (- π , 0). i.e. Put f (x ) = - ( − x ) 2 in (- π , 0) = - x 2 in (- π , 0) Now f (x) is odd in (- π , π ). 10
  • 11. ∴ f (x) = ∑b n =1 n sin nx ……………….(2) π π 2 2 bn = ∫ f ( x) sin nxdx = ∫ x 2 sin nxdx π 0 π 0 π 2   cos nx   sin nx   cos nx  = x2  −  − 2 x −  + 2  π  n   n 2   n 3  0  2 π 2   (−1) + 3 {(−1) − 1}  n +1 2 = n π n n   2 π 2 4    − , if n is odd = π  n n 3  − 2π , if n is even  n Using this value in(2), we get the half-range sine series of x 2 in (0 , π ). 2. Find the half-range sine series of f (x) = sin ax in (0 , l). Solution: We give an odd extension for f (x) in (-l , 0). i.e. we put f (x) = -sin[a(-x)] = sin ax in (-l , 0) ∴ f (x) is odd in (-l , l) ∞ nπx Let f (x ) = ∑b n =1 n sin l nπx l 2 bn = ∫ sin ax. sin l dx l 0 1   nπ  nπ l    = ∫ cos l − a  x − cos l + a  x dx l 0      l   nπ   nπ   1  sin  l − a  x sin  l + a  x  =    −    l   nπ   nπ     l − a  + a      l  0 1 1 = (−1) n +1 sin ( nπ − al ) − sin ( nπ + al ) nπ − al nπ + al 11
  • 12. 1 1 = (−1) n +1 sin al + ( −1) n +1 sin al nπ − al nπ + al 2nπ = (−1) n +1 sin al. 2 2 n π − a 2l 2 Using this values in (1), we get the half-range sine series as ∞ (−1) n +1 .n nπx sin ax = 2π sin al ∑ 2 2 sin n =1 n π − a l 2 2 l 3. Find the half-range cosine series of f (x ) = a in (0 , l). Deduce the sum of 1 1 1 2 + 2 + 2 + ∞ . 1 3 5 Solution: Giving an odd extension for f (x) in (-l , 0), f (x ) is made an odd function in (-l , l). nπx ∴ Let f(x) = ∑b n sin l ..……………(1) nπx l 2 bn = ∫ a sin dx l 0 l l  nπx   − cos l  = 2a   nπ l    = 2a nπ 1 − ( − 1) n { }    l 0   4a  , if n is odd =  nπ 0,  if n is even Using this value in (1), we get 4a ∞ 1 nπx a= ∑5 n sin l in (0 , l ) π n =1,3, Since the series whose sum is required contains constant multiples of squares of bn , we apply Parseval’s theorem. l 1 1 ∑ bn = l ∫ [ f ( x)] dx 2 2 2 0 12
  • 13. 1 16a 2 1 i.e. . 2 π2 ∑ ( 2n − 1) n =1, 3, 5 2 = a2 ∞ 8a 2 1 i.e. π2 ∑ ( 2n − 1) n =1 2 = a2 ∞ 1 π2 ∴ ∑ ( 2n − 1) 2 8 . n =1 = 4. Expand f (x) = x - x 2 as a Fourier series in -1 < x < 1 and using this series find the r.m.s. value of f (x ) in the interval. Solution: The Fourier series of f (x ) in (-1 , -1) is given by ∞ ∞ ao f (x) = 2 + ∑ an cos nπx + ∑ bn sin nπx n =1 n =1 .………………(1) 1 1 a o = ∫ f ( x)dx = ∫ ( x − x 2 ) dx 1 1 −1 −1 1  x2 x3  1 1  1 1 =  −  =  − − +   2   3  −1  2 3   2 3  −2 ao = ..........................(2) 3 1 1 ∫1 f ( x) cos nπx dx = −∫1( x − x ) cos nπx dx 1 an = 2 1− 1   sin nπx   − cos nπx   − sin nπx  = ( x − x 2 )   − (1 − 2 x )  2  + (−2) 3    n   n   n  −1 − cos nπ 3 cos nπ = − n2 n2 4 cos nπ an = − ……………….(3) n2 13
  • 14. 1 1 ∫1 f ( x) sin nπx dx = −∫1( x − x ) sin nπx dx 1 bn = 2 1− 1   − cos nπx   − sin nπx   cos nπx  = ( x − x 2 )   − (1 − 2 x )   + (−2) 3 3   nπ  nπ  n π  −1 2 2    − 2 cos nπ 2 cos nπ 2 cos nπ = − + 3 3 n 3π 3 nπ nπ n +1 2(−1) bn = ..........................( 4) nπ Substituting (2), (3), (4) in (1) we get 1 ∞ 4(−1) n +1 ∞ 2(−1) n +1 f (x) = − + ∑ cos nπx + ∑ sin nπx 3 n =1 n 2 n =1 nπ We know that r.m.s. value of f(x) in (-l , l) is 1 2 1 ∞ 1 ∞ a o + ∑ a n + ∑ bn 2 2 2 y = ……………….(5) 4 2 n =1 2 n =1 From (2) we get −2 2 4 ao = ⇒ ao = .………………..(6) 3 9 From (3) we get 4( −1) n +1 2 16 an = 2 ⇒ an = 4 ………………..(7) n n From (4) we get 2(−1) n +1 2 4 bn = ⇒ bn = 2 2 ..………………(8) nπ nπ Substituting (6), (7) and (8) in (5) we get 1 1 ∞  16 4  + ∑ 4 + 2 2  2 y = 9 2 n =1  n nπ  5. Find the Fourier series for f (x) = x 2 in − π < x < π . Hence show that 1 1 1 π4 + 4 + 4 + = 14 2 3 90 Solution: The Fourier series of f (x ) in (-1 , 1) is given by π2 ∞ 4(−1) n f (x) = 3 + ∑ n 2 cos nx n =1 14
  • 15. The co-efficients a o , a n , bn are 2π 2 4(−1) n ao = , an = , bn = 0 3 n2 Parseval’s theorem is (a ) π ∞ 1 1 2 1 ∫ [ f ( x)] dx = ao + ∑ 2 2 2 n + bn 2π −π 4 2 n =1  ao 2 1 ∞ 2  ( ) π ∫ [x ] + ∑ a n + bn  2 2 dx = 2π  2 ∴ −π  4  2 n =1   π  x5   π 4 1 ∞ 16  i.e.,   5  = 2π   + ∑ 4   −π  9 2 n =1 n  2π 5 2π 5 ∞ 16 i.e., − =π∑ 4 5 9 n =1 n 8π 4 ∞ 16 =∑ 4 45 n =1 n ∞ 1 π4 i.e., ∑ n 4 = 90 n =1 1 1 1 π4 i.e., + 2 + 2 + ∞ = 12 3 5 90 HARMONIC ANALYSIS The process of finding the Fourier series for a function given by numerical value is known as harmonic analysis. In harmonic analysis the Fourier coefficients ao , a n , and bn of the function y = f (x) in (0 , 2 π ) are given by a o = 2[mean value of y in (0 , 2 π )] a n = 2[mean value of y cos nx in (0 , 2 π )] bn = 2[mean value of y sin nx in (0 , 2 π )] (i) Suppose the function f (x) is defined in the interval (0 , 2l), then its Fourier series is, ao ∞ nπx ∞ nπx f (x) = 2 + ∑a n =1 n cos l + ∑ bn sin n =1 l and now, a o = 2[mean value of y in (0 , 2l)]  nπx  a n = 2 mean value of y cos in (0 , 2l )  l  15
  • 16. nπx  bn = 2 mean value of y sin in (0 , 2l )  l  (ii) If the half range Fourier sine series of f (x) in (0 , l) is, ∞ nπx f (x) = ∑b n =1 n sin l , then  nπx  bn = 2 mean value of y sin in (0 , l )  l  (iii) If the half range Fourier sine series of f (x) in (0 , π ) is, ∞ nπx f (x) = ∑b n =1 n sin l , then bn = 2[ mean value of y sin nx in (0 , π )] (iv) If the half range Fourier cosine series of f (x) in (0 , l) is, ao ∞ nπx f (x) = + ∑ a n cos , then 2 n =1 l a o = 2[mean value of y in (0 , l)]  nπx  a n = 2 mean value of y cos in (0 , l )  l  (v) If the half range Fourier cosine series of f (x) in (0 , π ) is, ao ∞ nπx f (x) = 2 + ∑a n =1 n cos l , then a o = 2[mean value of y in (0 , π )] a n = 2[ mean value of y cos nx in (0 , π )] . EXAMPLES 1. The following table gives the variations of a periodic function over a period T. 0 T T T 2T 5T T x 6 3 2 3 6 f (x) 1.98 1.3 1.05 1.3 -0.88 -0.25 1.98 2πx Show that f (x ) = 0.75 + 0.37 cos θ +1.004 sin θ , where θ = T Solution: Here the last value is a mere repetition of the first therefore we omit that value and consider the remaining 6 values. ∴ n = 6. 16
  • 17. 2πx Given θ= ..………………..(1) T T T T 2T 5T π 2π ∴ when x takes the values of 0, , , , , θ takes the values 0, , , 6 3 2 3 6 3 3 4π 5π π, , . (By using (1)) 3 3 Let the Fourier series be of the form ao f ( x) = + a1 cos θ + b1 sin θ , ………………(2) 2 ∑y where a o = 2 ,  n     ∑ y cos θ  a1 = 2 ,  n     ∑ y sin θ  b1 = 2 , n=6  n    θ y cos θ sin θ y cos θ y sin θ 0° 1.98 1.0 0 1.98 0 π 1.30 0.500 0.866 0.65 1.1258 3 2π 3 1.05 -0,500 0.866 -0.525 0.9093 π 1.30 -1 0 -1.3 0 4π 3 -0.88 -0.500 -0.866 0.44 0.762 5π 3 -0.25 0.500 -0.866 -0.125 0.2165 4.6 1.12 3.013 ∑y a o = 2  = 1.5, a1 = 2 ∑ y cos θ = 0.37  6  6   2 b1 = ∑ y sin θ = 1.00456 6 Substituting these values of a o , a1 , and b1 in (2), we get ∴ f (x) = 0.75 + 0.37 cos θ + 1.004 sin θ 2. Find the Fourier series upto the third harmonic for the function y = f (x) defined in (0 , π ) from the table x 0 π 2π 3π 4π 5π π 6 6 6 6 6 17
  • 18. f (x) 2.34 2.2 1.6 0.83 0.51 0.88 1.19 Solution: We can express the given data in a half range Fourier sine series. f ( x) = b1 sin x + b2 sin 2 x + b3 sin 3 x ..………………...(1) x y = f(0) sin x sin 2x sin 3x y sin x y sin 2x y sin 3x 0 2.34 0 0 0 0 0 0 30 2.2 0.5 0.87 1 1.1 1.91 2.2 60 1.6 0.87 0.87 0 1.392 1.392 0 90 0.83 1 0 -1 0.83 0 -0.83 120 0.51 0.87 -0.87 0 0.44 -0.44 0 150 0.88 0.5 -0.87 1 0.44 0.76 0.88 180 1.19 0 0 0 0 0 0 4.202 3.622 2.25  ∑ y sin x  1 Now b1 = 2   = [ 4.202] = 1.40   6  3   ∑ y sin 2 x  1 b2 = 2   = [ 3.622] = 1.207   6  3   ∑ y sin 3 x  1 b3 = 2   = [ 2.25] = 0.75   6  3  Substituting these values in (1), we get f (x) = 1.4 sin x + 1.21 sin 2x + 0.75 sin 3x 3. Compute the first two harmonics of the Fourier series for f(x) from the following data x 0 30 60 90 120 150 180 f (x ) 0 5224 8097 7850 5499 2626 0 Solution: Here the length of the interval is π . ∴ we can express the given data in a half range Fourier sine series i.e., f ( x) = b1 sin x + b2 sin 2 x ………………………(1) 18
  • 19. x y sin x sin 2x 0 0 0 0 30 5224 .5 0.87 60 8097 0.87 0.87 90 7850 1 0 120 5499 0.87 -0.87 150 2626 0.5 -0.87  ∑ y sin x  Now b1 = 2  = 7867.84   6    ∑ y sin 2 x  b2 = 2   = 1506.84   6   ∴ f (x) = 7867.84 sin x + 1506.84 sin 2x 4. Find the Fourier series as far as the second harmonic to represent the function given in the following data. x 0 1 2 3 4 5 f (x ) 9 18 24 28 26 20 Solution: Here the length of the interval is 6 (not 2 π ) i.e., 2l = 6 or l = 3 ∴ The Fourier series is ao πx 2πx πx 2πx f ( x) = + a1 cos + a 2 cos + b1 sin + b2 sin …………………..(1) 2 3 3 3 3 πx 2πx πx πx 2πx 2πx y cos y sin y cos y sin x 3 3 y 3 3 3 3 0 0 0 9 9 0 9 0 1 π 3 2π 3 18 9 15.7 -9 15.6 2 2π 3 4π 3 24 -12 20.9 -24 0 3 π 2π 28 -28 0 28 0 4 4π 3 8π 3 26 -13 -22.6 -13 22.6 5 5π 3 10 π 3 20 10 -17.4 -10 -17.4 125 -25 -3.4 -19 20.8 19
  • 20.  ∑ y  2(125) Now a o = 2 = = 41.66,  6  6   2 πx a1 = ∑ y cos = −8.33 6 3 2 πx b1 = ∑ y sin = −1.13 6 3 2 2πx a2 = 6 ∑ y cos 3 = −6.33 2 2πx b2 = 6 ∑ y sin 3 = 6.9 Substituting these values of a o , a1 , b1 , a 2 and b2 in (1), we get 41.66 πx 2πx πx 2πx f ( x) = − 8.33 cos − 6.33 cos − 1.13 sin + 6.9 sin 2 3 3 3 3 COMPLEX FORM OF FOURIER SERIES ∞ The equation of the form f ( x) = ∑c e n = −∞ n inπx l is called the complex form or exponential form of the Fourier series of f (x) in (c , c+2l). The coefficient c n is given by c + 2l 1 ∫ f ( x )e −inπx l cn = dx 2l c When l = π , the complex form of Fourier series of f (x) in (c , c+2 π ) takes the form ∞ f ( x) = ∑c e n = −∞ n inx , where c + 2π 1 ∫ f ( x )e −inx cn = dx. 2π c PROBLEMS 1. Find the complex form of the Fourier series of f (x) = e x in (0 , 2). Solution: Since 2l = 2 or l = 1, the complex form of the Fourier series is ∞ f ( x) = ∑c e n = −∞ n inπx 20
  • 21. 2 1 c n = ∫ f ( x)e −inπx dx 20 2 1 = ∫ e x e −inπx dx 20 2 1  e ( 1−inπ ) x  =   2  1 − inπ  0 = 1 2(1 − inπ ) {e 2(1−inπ ) − 1} = (1 + inπ ) {e ( cos 2nπ − i sin 2nπ ) − 1} 2 2(1 + n π 2 2 ) = (e 2 − 1)(1 + inπ ) 2(1 + n 2π 2 ) Using this value in (1), we get  e 2 − 1  ∞ (1 + inπ ) inπx  2  ∑ (1 + n 2π 2 ) e ex =     n =−∞ 2. Find the complex form of the Fourier series of f (x) = sin x in (0 , π ). Solution: Here 2l = π or l = π 2 . ∴ The complex form of Fourier series is ∞ f ( x) = ∑c e n = −∞ n i 2 nx …………………..(1) π 1 c n = ∫ sin xe −i 2 nx dx π 0 π 1  e −i 2 nx  =  { − i 2n sin x − cos x}  π 1 − 4n 2 0 = 1 π ( 4n − 1) 2 [ − e i 2 nx − 1 = − ] 2 π ( 4n 2 − 1) Using this value in (1), we get 2 ∞ 1 sin x = − ∑ 4n 2 − 1 .e i 2nx π n =−∞ in (0 , π ) 3. Find the complex form of the Fourier series of f (x) = e − ax in (-l , l). Solution: 21
  • 22. Let the complex form of the Fourier series be ∞ f ( x) = ∑c e n = −∞ n inπx l l 1 c n = ∫ f ( x)e −inπx l dx 2l −l l 1 = ∫ e − ax e −inπx l dx 2l −l l 1 = ∫ e −( al +inπ ) x / l dx 2l −l l 1  e −( al +inπ ) x l  =   2l  − ( al + inπ ) l  −l = 1 2( al + inπ ) [ e −( al +inπ ) − e ( al +inπ ) ] = 2 1 ( al + inπ ) [ e al (−1) n − e − al (−1) n ] [ e ± inπ = cos nπ ± i sin nπ = (−1) n ] sinh al (−1) n = al + inπ sinh al.( al − inπ ) (−1) n = a 2 l 2 + n 2π 2 Using this value in (1), we have ∞ (−1) n ( al − inπ ) inπx l e − ax = sinh al ∑ 22 2 2 e n = −∞ a l + n π in (-l , l) 4. Find the complex form of the Fourier series of f (x) = cos ax in (- π , π ), where a is neither zero nor an integer. Solution: Here 2l = 2 π or l = π . ∴ The complex form of Fourier series is ∞ f ( x) = ∑c e n = −∞ n inx ………………….(1) 22
  • 23. π 1 ∫π cos ax.e − inx cn = dx 2π − π 1  e −inx  =  2 { − in cos ax + a sin ax}  2π  a − n 2  −π = 1 2π ( a − n 2 ) 2 [ e −inπ ( − in cos aπ + a sin aπ ) − e inπ ( − in cos aπ − a sin aπ ) ] 1 = (−1) n 2a sin aπ 2π ( a − n ) 2 2 Using this value in (1), we get a sin aπ ∞ (−1) n inx cos ax = π ∑ n = −∞ a2 − n2 e in (- π , π ). UNIT 2 PART – A 1. Determine the value of a n in the Fourier series expansion of f ( x) = x 3 in − π < x < π . Ans: f ( x) = x 3 is an odd function. ∴ an = 0 2. Find the root mean square value of f ( x) = x 2 in the interval (0 , π ) . Ans: RMS Vale of f ( x ) = x 2 in (0 , π ) is π π π 1  x5  ∫ [x ] 2 1 2 2 1 y = dx = ∫ x 4 dx =   π 0 π 0 π 5   0 1 π 5  π 4 = = π5  5   3. Find the coefficient b5 of cos 5 x in the Fourier cosine series of the function f ( x ) = sin 5 x in the interval (0 , 2π ) Ans: Here f ( x) = sin 5 x Fourier cosine series is 23
  • 24. ao f (x) = 2 + ∑a n =1 n cos nx , where π π 2 2 an = ∫ f ( x) cos nx dx = π ∫ sin 5 x cos nx dx π 0 0 π 2 = 2π ∫ [ sin(5 + n) x + sin(5 − n) x] dx 0 π − 1  cos(5 + n) x cos(5 − n) x  = + =0 π  5+n  5 − n 0  cos x, if 0 < x < π 4. If f ( x) =  and f ( x) = f ( x + 2π ) for all x, find the sum of the Fourier 50, if π < x ≤ 2π series of f (x ) at x = π . Ans: Here π is a point of discontinuity. ∴ The sum of the Fourier series is equal to the average of right hand and left hand limit of the given function at x = π . f (π − 0) + f (π + 0) i.e., f (π ) = 2 cos π + 50 49 = = 2 2 5. Find bn in the expansion of x 2 as a Fourier series in (−π , π ) . Ans: bn = 0 Since f ( x) = x 2 is an even function in (−π , π ) . 6. If f (x) is an odd function defined in (-l , l) what are the values of a 0 Ans: a0 = 0 a n = 0 since f (x) is an odd function. 7. Find the Fourier constants bn for x sin x in (−π , π ) . Ans: bn = 0 Since f ( x) = x sin x is an even function in (−π , π ) . 24
  • 25. 8. State Parseval’s identity for the half-range cosine expansion of f (x ) in (0 , 1). Ans: 1 2 ∞ a0 2 ∫ [ f ( x)] dx = + ∑ an 2 2 0 2 n =1 where 1 a 0 = 2 ∫ f ( x) dx 0 1 a n = 2 ∫ f ( x) cos nx dx 0 9. Find the constant term in the Fourier series expansion of f ( x ) = x in (−π , π ) . Ans: a 0 = 0 since f (x ) is an odd function in (−π , π ) . 10. State Dirichlet’s conditions for Fourier series. Ans: (i) f (x) is defined and single valued except possibly at a finite number of points in (−π , π ) . (ii) f (x) is periodic with period 2 π . (iii) f (x) and f ′(x) are piecewise continuous in (−π , π ) . Then the Fourier series of f (x ) converges to (a) f (x) if x is a point of continuity f ( x + 0) + f ( x − 0) (b) if x is a point of discontinuity. 2 11. What you mean by Harmonic Analysis? Ans: The process of finding the Fourier series for a function given by numerical value is known as harmonic analysis. In harmonic analysis the Fourier coefficients ao , a n , and bn of the function y = f (x) in (0 , 2 π ) are given by a o = 2[mean value of y in (0 , 2 π )] a n = 2[mean value of y cos nx in (0 , 2 π )] bn = 2[mean value of y sin nx in (0 , 2 π )] 25
  • 26.  2x 1 + π , − π < x < 0  12. In the Fourier expansion of f ( x) =  in (−π , π ) . Find the value of bn , 1 − 2 x , 0 < x < π  π  the coefficient of sin nx. Ans: Since f (x) is an even function the value of bn = 0.  2( − x) 2x   In − π ≤ x ≤ 0 i.e., 0 ≤ − x ≤ π , f (− x ) = 1 − π = 1 + π = f ( x )   13. What is the constant term and the coefficient of cos nx, a n in the Fourier expansion of f ( x) = x − x 3 in (-7 , 7)? Ans: Given f ( x) = x − x 3 f ( − x ) = − x + x 3 = −( x − x 3 ) = − f ( x ) The given function is an odd function. Hence a 0 and a n are zero. 14. State Parseval’s identity for full range expansion of f (x ) as Fourier series in (0 , 2l). Ans: c + 2l 2 2 2 1 ∞ ∞ ∫ [ f ( x)] dx. = ao + ∑ a n + ∑ bn . 2 2l c 4 n =1 2 n =1 2 where c + 2l 1 nπx an = l ∫ c f ( x) cos l dx, n ≥ 0 c + 2l 1 nπx bn = l ∫ c f ( x) sin l dx, n ≥ 1 15. Find a Fourier sine series for the function f (x ) = 1; 0 < x < π . Ans: ∞ The Fourier sine series of f ( x) = ∑ bn sin nx …………………….(1) n =1 26
  • 27. π 2 bn = π ∫ f ( x) sin nx dx 0 π π 2  − cos nx  2 = ∫ sin nx dx =  π 0 π  n 0  =− 2 nπ ( (−1) n − 1) bn = 0, when ' n' is even 4 = , when ' n' is odd nπ ∞ 4 ∴ f ( x) = ∑ . sin nπ n =1, 3, 5, nπ 0 0< x<π 16. If the Fourier series for the function f ( x ) =  is sin x 0 < x < 2π − 1 2  cos 2 x cos 4 x  1 1 1 1 π −2 f ( x) = +  + +  + sin x Deduce that − + − ∞ = . π π  1.3 3.5  2 1.3 3.5 5.7 4 Ans: π Putting x = we get 2  π  −1 2  1 1 1  1 f = + − + − +  ∞ +  2  π π  1.3 3.5 5.7  2 −1 2  1 1 1  1 0= + − + − + ∞ + π π  1.3 3.5 5.7  2 1 1 1 π −2 − + − ∞ = . 1.3 3.5 5.7 4 17. Define Root mean square value of a function? Ans: c +2 l 1 ∫y 2 If a function y = f (x ) is defined in (c , c+2l), then dx is called the root mean- 2l c square(R.M.S.) value of y in (c , c+2l) and is denoted by y. 27
  • 28. c + 2l 2 1 Thus y = ∫y 2 dx. 2l c 18. If f ( x) = x 2 + x is expressed as a Fourier series in the interval (-2 , 2), to which value this series converges at x = 2. Ans: Since x = 2 is a point of continuity, the Fourier series converges to the arithmetic mean of f (x) at x = -2 and x = 2 f (2) + f (−2) 4 − 2 + 4 + 2 i.e., = =4 2 2 19. If the Fourier series corresponding to f ( x ) = x in the interval (0 , 2π ) is a0 ∞ + ∑ (a n cos nx + bn sin nx), without finding the values of a 0, a n , bn find the value of 2 n =1 2 ∞ a0 + ∑ (a n + bn ). 2 2 2 n =1 Ans: By using Parseval’s identity, 2 2π 2π a0 ∞ 1 1  x3  8 + ∑ (a n + bn ) = ∫ x 2 dx =   = π 2 . 2 2 2 π 0  3  π  0 3 n =1 20. Find the constant term in the Fourier series corresponding to f ( x) = cos 2 x expressed in the interval (−π , π ) . Ans: Given f ( x) = cos 2 x π π π 1 1  1 + cos 2 x  1 sin 2 x  Now a 0 = ∫π cos x dx = π −∫π  2 dx = π  x + 2  0 = 1 2 π −     PART B 1. (i) Express f ( x) = x sin x as a Fourier series in 0 ≤ x ≤ 2π . 28
  • 29. 2l  πx 1 2πx 1 3πx  (ii) Show that for 0 < x <l, x =  sin − sin + sin  . Using root mean square p l 2 l 3 l  1 1 1 value of x, deduce the value of 2 + 2 + 2 + 1 2 3 2. (i) Find the Fourier series of periodicity 3 for f ( x ) = 2 x − x 2 in 0 < x < 3. (ii) Find the Fourier series expansion of period 2 π for the function y = f (x) which is defined in (0 , 2π ) by means of the table of values given below. Find the series upto the third harmonic. x 0 π 2π π 4π 5π 2π 3 3 3 3 f (x ) 1.0 1.4 1.9 1.7 1.5 1.2 1.0 3.(i) Find the Fourier series of periodicity 2 π for f ( x) = x 2 for 0 < x < 2 π . l 4l  πx 1 3πx  (ii) Show that for 0 < x <l, x = −  cos + 2 cos +  . Deduce that 2 π2  l 3 l  1 1 1 π4 + 4 + 4 + = . 14 3 5 96 l − x, 0 < x ≤ l 4. (i) Find the Fourier series for f ( x) =  . Hence deduce the sum to infinity of 0, l ≤ x ≤ 2l ∞ 1 the series ∑ (2n + 1) n =0 2 . (ii) Find the complex form of Fourier series of f ( x ) = e ax (−π < x < π ) in the form sinh aπ ∞ a + in inx π ∞ (−1) n e ax = π ∑ (−1) n −∞ a2 + n2 e and hence prove that =∑ 2 a sinh aπ −∞ n + a 2 . 5. Obtain the half range cosine series for f ( x) = x in (0 , π ). 6. Find the Fourier series for f ( x) = cos x in the interval (−π , π ) . 1 1 π3 7. (i) Expanding x(π − x) as a sine series in (0 , π ) show that 1 − + 3 + = . 33 5 32 (ii) Find the Fourier series as far as the second harmonic to represent the function given in the following data. x 0 1 2 3 4 5 29
  • 30. f (x ) 9 18 24 28 26 20 8. Obtain the Fourier series for f (x ) of period 2l and defined as follows  L + x in ( − L,0) f ( x) =   L − x in (0, L) 1 1 1 π2 Hence deduce that + 2 + 2 + = . 12 3 5 8 9. Obtain the half range cosine series for f ( x) = x in (0 , π ). 1 in (0, π ) 10. (i) Find the Fourier series of f ( x ) =  2 in (π ,2π ) (ii) Obtain the sine series for the function  l  x in 0 ≤ x ≤ 2  f ( x) =  l − x in l ≤ x ≤ l   2 11. (i) Find the Fourier series for the function 0 in (−1, 0) f ( x) =  and f ( x + 2) = f ( x ) for all x. 1 in (0, 1) (ii) Determine the Fourier series for the function πx, 0 ≤ x ≤1 f ( x) =  π (2 − x ), 1 ≤ x ≤ 2 12. Obtain the Fourier series for f ( x ) = 1 + x + x 2 in (−π , π ) . Deduce that 1 1 1 π2 + 2 + 2 + = . 12 2 3 6 13. Obtain the constant term and the first harmonic in the Fourier series expansion for f (x) where f (x) is given in the following table. x 0 1 2 3 4 5 6 7 8 9 10 11 f (x) 18. 18. 17. 15. 11. 8.3 6. 5. 6. 9. 12. 15.7 0 3 4 0 4 0 7 6 0 6 14. (i) Express f ( x) = x sin x as a Fourier series in (−π , π ). 30
  • 31. (ii) Obtain the half range cosine series for f ( x) = ( x − 2) 2 in the interval 0 < x < 2. 15. Find the half range sine series of f ( x) = x cos x in (0 , π ). 16. (i) Find the Fourier series expansion of f (x ) = e − x in (−π , π ) (ii) Find the half-range sine series of f (x ) = sin ax in (0 , l). 17. Expand f (x ) = x - x 2 as a Fourier series in -1 < x < 1 and using this series find the r.m.s. value of f (x) in the interval. 18. The following table gives the variations of a periodic function over a period T. 0 T T T 2T 5T T x 6 3 2 3 6 f (x) 1.98 1.3 1.05 1.3 -0.88 -0.25 1.98 2πx Show that f (x ) = 0.75 + 0.37 cos θ +1.004 sin θ , where θ = T 19. Find the Fourier series upto the third harmonic for the function y = f (x) defined in (0 , π ) from the table x 0 π 2π 3π 4π 5π π 6 6 6 6 6 f (x) 2.34 2.2 1.6 0.83 0.51 0.88 1.19 20. (i) Find the half-range (i) cosine series and (ii) sine series for f (x ) = x 2 in (0 , π ) (ii) Find the complex form of the Fourier series of f (x) = cos ax in (- π , π ). 31